Sie sind auf Seite 1von 94

Compiled by:MANY DOCTORS........it will be selfish if one name is mentioned.............

this is a
compilation of work by the people who discussed in the thread All India 2011--explanatory
answer pool...STRICLTLY "NOT" FOR POINTLESS CHATTING
In RxPG ONLINE.com..
There are questions for which there were conflicting views....for detailed
discussion on those questions........u can check out the thread..........u can
contribute if u have any valuable suggestions too.......

ALL INDIA POST GRADUATE MEDICAL EXAMINATION


2011-MD/MS
1. Most common nerve injured in supracondylar fracture humerus?
A. Median
B. Radial
C. Ulnar
D. Anterior interosseus nerve
Ans: D (Anterior interosseus nerve)

The Mnemonic for the descending order of incidence of involvement of nerves


injured in Supracondylar fracture is AMRU
That is:
Anterior Interosseus Nerve> Median > Radial >Ulnar

2. Version 1:
Earliest symptom of GERD which becomes pathological in an infant is?
A. Respiratory distress
B. Upper GI bleed
C. Regurgitation
D. Intestinal obstruction

Ans: A (Respiratory distress)

Version 2: Most common symptom of GERD in an infant is?


A. Respiratory distress
B. Upper GI bleed
C. Regurgitation
D. Intestinal obstruction

Ans: C (Regurgitation)

Reference: Nelson Textbook Of Pediatrics 18th ed.

Infantile reflux manifests more often with regurgitation (especially


postprandially), signs of esophagitis (irritability, arching, choking, gagging,
feeding aversion), and resulting failure to thrive; symptoms resolve
spontaneously in the majority by 12–24 mo.
Older children, in contrast, may have regurgitation during the preschool
years; complaints of abdominal and chest pain supervene in later childhood and
adolescence.
Occasional children present with neck contortions (arching, turning of
head) designated Sandifer syndrome.
The respiratory presentations are also age dependent: GERD in infants
may manifest as obstructive apnea or as stridor or lower airway disease
in which reflux complicates primary airway disease such as
laryngomalacia or bronchopulmonary dysplasia. Otitis media, sinusitis,
lymphoid hyperplasia, hoarseness, vocal cord nodules, and laryngeal edema have
all been associated with GERD. In contrast, airway manifestations in older
children are more frequently related to asthma or to otolaryngologic disease such
as laryngitis or sinusitis.
3. Basal metabolic rate is closely associated with?
A. Lean body mass
B. Body surface area
C. Body mass index
D. Body weight
Ans: A Lean Body mass (REPEAT)
4. A girl presented with recurrent occipital headache associated with ataxia and
vertigo. There is history of Mother having similar complaints. Most probable
diagnosis is?
A. Vestibular neuronitis
B. Basillar migraine
C. TIA
D. –
Ans: B Basilar migraine (REPEAT)

5. Drug of choice for central Diabetes Insipidus is?


A. Desmopressin
B. Leuperolide
C. Thiazide diuretics
D. Lithium

Ans: A Desmopressin

Reference: Harrison Principles Of Internal Medicine 17th ed

The signs and symptoms of uncomplicated pituitary DI can be eliminated


completely by treatment with desmopressin (DDAVP), a synthetic analogue
of AVP .

It acts selectively at V2 receptors to increase urine concentration and decrease


urine flow in a dose-dependent manner .

It is also more resistant to degradation than AVP and has a three- to fourfold
longer duration of action.
Desmopressin (DDAVP) can be given by IV or SC injection, nasal inhalation, or
oral tablet.

6. A 32 year old mountaineer has a hematocrit of 70%. What is the possible


explanation?
A. Polycythemia with relative dehydration
B. High altitude cerebral oedema
C. High altitude pulmonary oedema
D. Hemodilution

Ans.A Polycythemia and relative dehydration

Reference: Harrison Principles Of Internal Medicine 17th ed

Polycythemia can be spurious (related to a decrease in plasma


volume; Gaisbock's syndrome), primary, or secondary in origin.

The secondary causes are all associated with increases in EPO levels: either a
physiologically adapted appropriate elevation based on tissue hypoxia (lung
disease, high altitude, CO poisoning, high-affinity hemoglobinopathy) or an
abnormal overproduction (renal cysts, renal artery stenosis, tumors with ectopic
EPO production). A rare familial form of polycythemia is associated with normal
EPO levels but hyperresponsive EPO receptors due to mutations

7. Tetracycline is used in the prophylaxis of which of the following diseases?


A. Cholera
B. Brucellosis
C. Leptospirosis
D. Meningitis

Ans: A Cholera

Reference: Goodman & Gilman's Pharmacology ; Harrison Principles Of Internal


Medicine 17th ed;Internet

Doxycycline (300 mg as a single dose) is effective in reducing stool


volume and eradicating Vibrio cholerae from the stool within 48 hours.
Antimicrobial agents, however, are not substitutes for fluid and electrolyte
replacement in this disease.

In addition, some strains of V. cholerae are resistant to tetracyclines.

Tetracycline is used in chemoprophylaxis of Cholera.

Doxycycline is used in chemoprophylaxis of Leptospirosis.

8. A 7 month old child presents with complaints of recurrent bouts of cough.


What is the best way to confirm the diagnosis?
A. Nasophayngeal swab
B. Cough plate culture
C. Tracheal aspirate
D. –

Ans: A Nasopharyngeal swab

Harrison 17th ed

The diagnosis is Pertusis and we have to find the best specimen to confirm the
diagnosis.

Harrison Says:
Culture of nasopharyngeal secretions remains the gold standard of
diagnosis, although DNA detection by polymerase chain reaction (PCR) is
replacing culture in many laboratories because of increased sensitivity and
quicker results. The best specimen is collected by nasopharyngeal
aspiration, in which a fine flexible plastic catheter attached to a 10-mL syringe is
passed into the nasopharynx and withdrawn while gentle suction is applied.

Lets look as what Jawetz Microbiology has go to say on the topic:

SPECIMENS
A saline nasal wash is the preferred specimen. Nasopharyngeal swabs or
cough droplets expelled onto a "cough plate" held in front of the patient's mouth
during a paroxysm are sometimes used but are not as good as the saline nasal
wash.

Lets look into CMDT 2011 views:


The diagnosis is established by isolating the organism from
nasopharyngeal culture. A special medium (eg, Bordet-Gengou agar) must be
requested.
Well with these reference ideally the answer is none. But from all references
Nasopharyngeal swab seemed more appropriate.

9. Aflatoxin is produced by?


A. Aspergillus flavus
B. Aspergillus niger
C. Candida
D. –

Ans: A Aspergillus Flavus

Reference: Jawetz Microbiology

MYCOTOXINS
Many fungi produce poisonous substances called mycotoxins that can cause acute
or chronic intoxication and damage. The mycotoxins are secondary metabolites,
and their effects are not dependent on fungal infection or viability. A variety of
mycotoxins are produced by mushrooms (eg, amanita species), and their
ingestion results in a dose-related disease called mycetismus. Cooking has little
effect on the potency of these toxins, which may cause severe or fatal damage to
the liver and kidney. Other fungi produce mutagenic and carcinogenic compounds
that can be extremely toxic for experimental animals. One of the most potent
is aflatoxin, which is elaborated by Aspergillus flavus and related molds
and is a frequent contaminant of peanuts, corn, grains, and other foods.

10. Most important prognostic factor in congenital diaphragmatic hernia?


A. Pulmonary hypertension
B. Timing of surgery
C. Size of defect
D. Gestational Age

Ans: A Pulmonary hypertension

Reference: Nelson Textbook Of Pediatrics18th ed

Relative predictors of a poor prognosis include an associated major


anomaly, symptoms before 24 hr of age, severe pulmonary hypoplasia,
herniation to the contralateral lung, and the need for ECMO.
Serious sequelae include pulmonary function changes,
neurodevelopmental delays, and growth retardation.
Pulmonary problems continue to be a source of morbidity for long-
term survivors of CDH.
Children receiving CDH repair studied at 6–11 yr of age demonstrate
significant decreases in forced expiratory flow at 50% of vital capacity and
decreased peak expiratory flow. Both obstructive and restrictive patterns can
occur.
Those without severe pulmonary hypertension and barotrauma do
the best. Those at highest risk include children who required ECMO and patch
repair, but the data clearly show that non-ECMO CDH survivors also require
frequent attention to pulmonary issues. At discharge, up to 20% of infants
require oxygen, but only 1–2% require it past 1 yr of age.
11. A surgeon removed the part of liver to the left of the falciform ligament.
Which segments have been removed?
A. 1 & 4a
B. 2 & 3
C. 1 & 4b
D. 5 & 6

Ans: B 2 & 3(REPEAT)

Reference: Sabiston Surgery 18th ed

Resection of segments II and III is a commonly performed sublobar


resection and is often referred to as a left lateral segmentectomy and left lateral
sectionectomy or left lobectomy.Left lobe is that part of liver to the left of the
falciform ligament.

Extra edge:
Nomenclature for Most Common Major Anatomic Hepatic Resections
GOLDSMITH AND
SEGMENTS COUINAUD, 1957 WOODBURNE, 1957 BRISBANE, 2000
V-VIII Right hepatectomy Right hepatic lobectomy Right hemi-
hepatectomy
IV-VIII Right lobectomy Extended right hepatic Right
lobectomy trisectionectomy
II-IV Left hepatectomy Left hepatic lobectomy Left hemi-
hepatectomy
II, III Left lobectomy Left lateral segmentectomy Left lateral
sectionectomy
II, III, IV, V, Extended left Extended left lobectomy Left trisectionectomy
VIII hepatectomy

12. Punnett square is used for


A.Finding genotype of offspring
B.Statistical analysis
C–
D–

Ans:A Finding the genotype of the offspring

Reference: The Tennessee Gateway Science

Punnett square is a Diagram used to identify possible combinations of recessive


and dominant alleles in OFFSPRING.To create a punnette square divide a suare
into 4 parts and write the letters that represent the alleles of one parent on top of
the suare and the alleles of the other parent on the side of the square..Just like
we make a 2X2 table for statistics questions.Combine the allele of one parent
with the other and work out the 4 possible combinations.The punnett square does
not give the exact information about the offspring but instead gives the
probability.

Have a look at this diagram...its a simple concept that we all have been practising
since childhood but may not have been aware of the name of this method...
13. Cavitation in lungs is seen in?
A. Mycolplasma pneumonia
B. Primary Tuberculous
C. Streptococcal pneumonia
D.Staphylococcal pneumonia

Ans: D Staphylococcal Pneumonia (REPEAT)

14. All are false with respect to PSUDOHYPOPARATHYROIDISM except?


a. Decreased cAMP response
b. Decreased IP3 response
c. Increased GTPase activity
d.Gain of function mutation of Gs alpha subunit

Ans:A Decreased cAMP

Reference: Harrison Principles Of Internal Medicine17th ed

PSEUDOHYPOPARATHYROIDISM:

PHP is a hereditary disorder characterized by symptoms and signs of


hypoparathyroidism, typically in association with distinctive skeletal and
developmental defects. The hypoparathyroidism is due to a deficient end-organ
response to PTH.

Hyperplasia of the parathyroids, a response to hormone resistance, causes


elevation of PTH levels.
Individuals with Pseudohypoparathyroidism 1, the most common of the disorders,
show a deficient urinary cyclic AMP response to administration of
exogenous PTH.

Patients with PHP-I are divided into type a, with Albright’s Hereditary
Osteodystrophy(AHO) and reduced amounts of Gsalpha in vitro assays
witherythrocytes, and type b, lacking AHO and with normal amounts of Gsalpha in
erythrocytes. There is a third type (PHP-Ic, reported in a few patients) that differs
from PHP-Ia only in having normal erythrocyte levels of Gsalpha despite having
AHO, hypocalcemia, and decreased urinary cyclic AMP responses to PTH
(presumably with a post-Gsalpha defect in adenyl cyclase stimulation).

15. Difference between follicular carcinoma and follicular adenoma is?


A. Vascular invasion
B. Mitosis
C. Nuclear pleomorphism
D. Hurthle cells
Ans: A Vascular invasion (REPEAT)
16. Antidepressant drug that can be used in nocturnal eneuresis?
A. Imipramine
B. Fluoxamine
C.
D.
Ans: A Imipramine
Reference: Kaplan & Sadock's Synopsis of Psychiatry
Imipramine is efficacious and has been approved for use in treating
childhood enuresis, primarily on a short-term basis. Initially, up to 30 percent of
patients with enuresis stay dry, and up to 85 percent wet less frequently than
before treatment.
The success often does not last, however, and tolerance can develop after
6 weeks of therapy. Once the drug is discontinued, relapse and enuresis at
former frequencies usually occur within a few months. The drug's adverse effects,
which include cardiotoxicity, are also a serious problem.

17. A 40yr old female patient presented with depressed mood, loss of appetite
and no interest in surroundings for the past 1yr. There is associated insomnia.
These symptoms followed soon after a business loss 1yr back.Which of the
following statements is true regarding the management of this patient?
A. No treatment is necessary as it is due to business loss
B. SSRI is the most efficacious of the available drugs
C. Antidepressant treatment is based on the side effect profile of the drugs
D. Combination therapy of 2 anti depressant drugs

Ans:C Treatment is started based on the side effect profile

Reference: Kaplan & Sadock's Synopsis of Psychiatry

Initial Medication Selection

The available antidepressants do not differ in overall efficacy, speed of


response, or long-term effectiveness. Antidepressants, however, do differ in their
pharmacology, drug-drug interactions, short- and long-term side effects,
likelihood of discontinuation symptoms, and ease of dose adjustment. Failure to
tolerate or to respond to one medication does not imply that other medications
will also fail. Selection of the initial treatment depends on the chronicity of
the condition, course of illness (a recurrent or chronic course is associated with
increased likelihood of subsequent depressive symptoms without treatment),
family history of illness and treatment response, symptom severity, concurrent
general medical or other psychiatric conditions, prior treatment responses to
other acute phase treatments, potential drug–drug interactions, and
patient preference.
In general, approximately 45 to 60 percent of all outpatients with
uncomplicated (i.e., minimal psychiatric and general medical comorbidity),
nonchronic, nonpsychotic major depressive disorder who begin treatment with
medication respond (i.e., achieve at least a 50 percent reduction in baseline
symptoms); however, only 35 to 50 percent achieve remission (i.e., the virtual
absence of depressive symptoms).

18. An Infant is brought to the casualty with reports of violent shaking by parents.
Most characteristic injury is?
A. Long bone fracture
B. Ruptured spleen
C. Subdural hematoma
D. Skull bone fracture

Ans:C Subdural hematoma

Reference: Forensic Medicine By Dr.Reddy

Under the Topic Battered Baby syndrome Reddy talks about SUBDURAL
HEMATOMA being the MOST CHARACTERISTIC FEATURE of violent shaking of an
infant by the Parent...Its called INFANTILE WHIPLASH SYNDROME....In Battered
baby syndrome multiple Long bone fractures at various stages of healing may be
seen and not in infantile whiplash syndrome..

19. Gun powder on clothing can be visualized by?


A. Magnifying lens
B. UV rays
C. Infrared rays
D. Dye/spray

Ans:C IR rays

20. All have polysaccharide capsule related antigen antibody response except?
(Reframed by Dr.jammroy)
A. Neisseria meningitidis
B. Pneumococcus
C. Bordetella pertussis
D. Haemophilus influenza

Ans: C Bordetella pertusis

Reference: Harrison 17th ed

Pathogenetic factors in Bordetella :


B. pertussis produces a wide array of toxins and biologically active
products that are important in its pathogenesis and in immunity. Most of these
virulence factors are under the control of a single genetic locus that regulates
their production, resulting in antigenic modulation and phase variation. Although
these processes occur both in vitro and in vivo, their importance in the
pathobiology of the organism is unknown; they may play a role in intracellular
persistence and person-to-person spread.
The organism's most important virulence factor is pertussis toxin,
which is composed of a B oligomer–binding subunit and an enzymatically active A
protomer that ADP-ribosylates a guanine nucleotide-binding regulatory protein (G
protein) in target cells, producing a variety of biologic effects. Pertussis toxin has
important mitogenic activity, affects the circulation of lymphocytes, and serves as
an adhesin for bacterial binding to respiratory ciliated cells. Other important
virulence factors and adhesins are filamentous hemagglutinin, a component of
the cell wall, and pertactin, an outer-membrane protein. Fimbriae, bacterial
appendages that play a role in bacterial attachment, are the major antigens
against which agglutinating antibodies are directed. These agglutinating
antibodies have historically been the primary means of serotyping B. pertussis
strains. Other virulence factors include tracheal cytotoxin, which causes
respiratory epithelial damage; adenylate cyclase toxin, which impairs host
immune-cell function; dermonecrotic toxin, which may contribute to respiratory
mucosal damage; and lipooligosaccharide, which has properties similar to those
of other gram-negative bacterial endotoxins.

Immunity against Bordetella:


Both humoral and cell-mediated immunity are thought to be important in
pertussis. Antibodies to pertussis toxin, filamentous hemagglutinin, pertactin,
and fimbriae are all protective in animal models. Pertussis agglutinins were
correlated with protection in early studies of whole-cell pertussis vaccines. Serologic
correlates of protection conferred by acellular pertussis vaccines have not been
established, although antibody to pertactin, fimbriae, and (to a lesser degree) pertussis
toxin correlated best with protection in two efficacy trials. The duration of immunity after
whole-cell pertussis vaccination is short-lived, with little protection remaining after 10–
12 years. After a three-dose infant primary series of acellular pertussis vaccine,
protection persists for at least 5–6 years; the duration of immunity after a four- or five-
dose schedule is not yet known. Although immunity after natural infection has been said
to be lifelong, seroepidemiologic evidence suggests that it may not be and that
subsequent episodes of clinical pertussis are prevented by intermittent subclinical
infection.

21. Signature fracture refers to?


A. Depressed skull fracture
B. Suture displacement fracture
C. Contrecoup injury
D. Fracture at foramen magnum
Ans: Depressed fracture (REPEAT)
22. Which among the following is most frequently seen in anti phospholipid
antibody syndrome?(Pls CORRECT Q)
A. Beta 2 microglobulin antibody
B. Anti nuclear antibody
C. Anti centromere antibody
D. Anti beta 2 glycoprotein antibody
Ans: D Anti beta 2 GP Ab
The antiphospholipid antibody syndrome (APS) may be defined as the
occurrence of arterial or venous thrombosis or recurrent miscarriage in
association with laboratory evidence of persistent antiphospholipid antibody. The
antibody may manifest itself as either a lupus anticoagulant detected by clotting
tests or an anticardiolipin antibody (ACA) detected by immunoassay. The
syndrome may be associated with an autoimmune disorder, especially SLE
(secondary), or may occur independently of other autoimmune disorders
(primary).
A large proportion of antiphospholipid antibodies are actually directed
against complexes of phospholipid with protein, most notably beta-2-
glycoprotein I. The ACA immunoassay detects both these clinically significant
antibodies and antibodies directed purely against cardiolipin and not beta-2-
glycoprotein I. The latter antibodies are transient, and are often associated with
infection but not thrombosis. A positive anti-b2 GPI assay is evidence in favor of
APS, once persistence is demonstrated by repeating the assay after eight weeks.
23. What factor is responsible for deciding whether an antibody will remain
membrane bound or get secreted?
A. RNA splicing
B. Class switching
C. Differential RNA processing
D. Allelic exclusion
Ans: C Differential RNA Processing(REPEAT)
24. Blood Chimerism is associated with?
A. Monochorionic monoamniotic twins
B. Monochorionic diamniotic twins
C. Singleton pregnancy
D. Vanishing twin
Ans: B Monochorionic diamniotic twins (this is the answer if these were
the options)
Explanation: Reference:
“ Journal Ref: Blood Chimerism in a Dizygotic Dichorionic Pregnancy”
“Blood chimerism in monochorionic twins conceived by induced ovulation: Case
report”
Blood chimerism “is more common in monochorionic dizygotic twins and rare
in dichorionic twins
Cases in which a monochorionic placenta occurs in a twin pregnancy,vascular
anstomosis is well described and can lead to blood chimerism as well as twin twin
transfusion syndrome.
In contrast interplacental vascular communications occur only with very rare
exceptions in a dichorionic placenta”
25. In expectant management of placenta praevia, all are done except?
A. Cervical encirclage
B. Anti D
C. Corticosteroids
D. Blood transfusion
Ans: A Cervical encirclage
Reference: Williams Obstetrics
The treatment of classical cervical incompetence is cerclage. The operation
is performed to surgically reinforce
the weak cervix by some type of purse-string suturing. Bleeding, uterine
contractions, or ruptured membranes are usually contraindications for
cerclage.
26. Which drug is not in the conduct of labour in a woman with rheumatic heart
disease?
A. Methylergometrine
B. Carboprost
C. Syntocin
D. Misoprostol
Ans: A Methylergometrine(REPEAT)
27. Which is not an autoimmune disease?
A. SLE
B. Grave's disease
C. Myasthenia gravis
D. Sickle cell disease
Ans: D Sickle Cell Disease(Sickle cell disease is a hemoglobinopathy and
not an autoimmune disease)
28. All are true regarding selective estrogen receptor downregulator
(SERD),fulvestrant except?
A. Used for breast cancer
B. Is a selective oestrogen antagonist
C. Is slower acting, safer, LESS effective than SERM
D. Given as once a month im dose
Ans: C It’s slower acting and less efficacious than SERM
Reference: This Question has been taken line to line from Goodman Gilman
Pharmacology as u will see below:
Fulvestrant
Fulvestrant is the first FDA approved agent in the new class of
estrogen-receptor downregulators, which were hypothesized to have an
improved safety profile, faster onset, and longer duration of action than
the SERMs due to their pure ER antagonist activity .
Fulvestrant was approved in 2002 for postmenopausal women with
hormone receptor-positive metastatic breast cancer that has progressed
despite antiestrogen therapy.
Mechanism of Action
Fulvestrant is a steroidal antiestrogen that binds to the ER with an affinity more
than 100 times that of tamoxifen, inhibits its dimerization, and increases its
degradation.
Preclinical studies suggest that as a consequence of this ER "downregulation,"
ER-mediated transcription is abolished, completely suppressing the expression of
estrogen-dependent genes (Howell et al., 2004b). This difference in the activity of
fulvestrant likely explains why fulvestrant demonstrates efficacy against
tamoxifen-resistant breast cancer.
However, the hypothesis that fulvestrant provides more effective antiestrogen
activity than tamoxifen was not confirmed by a clinical trial comparing fulvestrant
(250 mg intramuscularly monthly) with tamoxifen (20 mg orally daily) as first-
line therapy in metastatic breast cancer.
Absorption, Fate, and Excretion
Maximum plasma concentrations are reached about 7 days after intramuscular
administration of fulvestrant and are maintained over a period of 1 month. The
plasma half-life is approximately 40 days. Steady-state concentrations are
reached after 3 to 6 monthly injections. There is extensive and rapid distribution,
predominantly to the extravascular compartment.
Various pathways, similar to those of steroid metabolism including oxidation,
aromatic hydroxylation, and conjugation, extensively metabolize fulvestrant.
CYP3A4 appears to be the only CYP isoenzyme involved in the oxidation of
fulvestrant. Several preclinical and clinical studies have confirmed that fulvestrant
is not subject to CYP3A4 interactions that might affect the safety or efficacy of
the drug. The putative metabolites possess no estrogenic activity and only the
17-keto compound demonstrates a level of antiestrogenic activity about 4.5 times
less than that of fulvestrant. The major route of excretion is via the feces, with
less than 1% being excreted in the urine.
Therapeutic Uses
Fulvestrant typically is administered as a 250-mg intramuscular injection at
monthly intervals. It is used in postmenopausal women as antiestrogen therapy
of hormone receptor-positive metastatic breast cancer after progression on first-
line antiestrogen therapy such as tamoxifen. Fulvestrant is at least as effective in
this setting as the third-generation aromatase inhibitor anastrozole.
Fulvestrant 250 mg (administered as a once-monthly 5-ml intramuscular
injection) also has been compared with tamoxifen 20 mg (orally once daily) in a
trial of postmenopausal women with ER-positive and/or progesterone receptor
(PR)-positive or ER/PR-unknown metastatic breast cancer who had not previously
received endocrine or chemotherapy. There was no difference between
fulvestrant and tamoxifen in time to disease progression in either the entire study
population or the subset of patients with ER- and/or PR-positive disease.
Observed differences in other efficacy endpoints favored tamoxifen, and
fulvestrant equivalence was not demonstrated. The long time to steady-state
plasma levels for fulvestrant has brought into question the results of existing
studies, and trials are in progress to test the relative efficacy of giving an initial
loading dose followed by regular monthly injections.
Clinical Toxicity
Fulvestrant generally is well tolerated with the most common adverse
events being nausea, asthenia, pain, vasodilation (hot flushes), and
headache. Injection site reactions, seen in about 7% of patients, are reduced by
giving the injection slowly. In the study comparing anastrozole and fulvestrant,
quality-of-life outcome measures were maintained over time with no significant
difference between the drugs .
29. A farmer developed swellings in the inguinal region which later ulcerated and
associated with constitutional symptoms. What stain can be used to detect the
bipolar stained organisms?
A. Albert's stain
B. Waysons stain
C. Ziehl neelsen stain
D. Nigrosin stain
Ans: B Wayson stain(REPEAT)
30. An 8 year old boy completed 8 out of 10 day course of cefaclor. Now he
developed a generalized erythmatic rash which is mildly pruritic and
associated lymphadenopathy. Diagnosis is?
A. Kawasaki disease
B. Type 3 hypersensitivity
C. Anaphylaxis
D. Infectious mononucleosis
Ans: B Type 3 Hypersensitivity (REPEAT)
31. Rave drug is?
A. Cannabis
B. Cocaine
C. Heroin
D. Esctasy
Ans: D Esctasy
32. How to differentiate ASD from VSD in X-ray?
A. Enlarged Left atrium
B. Normal left atrium
C. Pulmonary congestion
D. Aortic shadow
Ans: B Normal LA size
LA remains normal in ASD despite volume overload since it can decompress
through 2 outlets that is into RA and into LV.
So LA enlargement is not seen in ASD whereas it is seen in VSD.
33. Regarding an imbecile, all are true except?
A. IQ is 50-60 B. Intellectual capacity equivalent to a child of 3-7
years of age
C. Not able to take care of themselves
D. Condition is congenital or acquired at an early age
Ans: A IQ is 50-60
Reference: Internet sources.

IQ
Classification
Range
Borderline
70-80
deficiency
50-69 Moron
20-49 Imbecile
below 20 Idiot
34. Which is not true regarding diet modification recommended in high
cardiovascular risk group?
A. Cholesterol less then 100 mg/1000kcal/day
B. Avoid alcohol
C. Fat intake 10% of total calories
D. Salt limitation to less than 5 gm
Ans: C Fat intake <10% of total calories(NEEDS MORE DETAILS)
Reference: American Heart association 2006 guidelines on diet in cardiovascular
disease.
These guidelines confirm the options 1/2/4...regarding option 3....it is the
saturated fat content which should be less that 10%.the total fat intake should be
less than 30% of the overall calorie intake.
IF THE OPTION 3 WAS....Saturated Fat intake less than 10% then the answer
shall become Avoid alcohol...as AHA guidelines say upto 2-3 drink per day is
fine....
35. Maintenance dose of which of the following drugs is used worldwide for opioid
dependence?
A. Naltrexone
B. Methadone
C. lmipramine
D. Disulfiram
Ans: B Methadone
Reference: Kaplan and sadock’s Synopsis of Psychiatry
Methadone is a synthetic narcotic (an opioid) that substitutes for
heroin and can be taken orally. When given to addicts to replace their
usual substance of abuse, the drug suppresses withdrawal symptoms. A
daily dosage of 20 to 80 mg suffices to stabilize a patient, although daily doses of
up to 120 mg have been used. The duration of action for methadone exceeds 24
hours; thus, once-daily dosing is adequate. Methadone maintenance is continued
until the patient can be withdrawn from methadone, which itself causes
dependence. An abstinence syndrome occurs with methadone withdrawal, but
patients are detoxified from methadone more easily than from heroin. Clonidine
(0.1 to 0.3 mg three to four times a day) is usually given during the detoxification
period.
Methadone maintenance has several advantages. First, it frees persons with
opioid dependence from using injectable heroin and, thus, reduces the chance of
spreading HIV through contaminated needles. Second, methadone produces
minimal euphoria and rarely causes drowsiness or depression when taken for a
long time. Third, methadone allows patients to engage in gainful employment
instead of criminal activity. The major disadvantage of methadone use is that
patients remain dependent on a narcotic.
36. Best test/Gold standard test for assessing betaHCG function/action?
A. Radioimmunoassay
B. ELISA
C. Latex test
D. Bioassay
Ans: D Bioassay
37. Vitamin K is involved in the posttranswerlational modification of?
A. Glutamate
B. Aspartate
C. --
D. –
Ans: A Glutamate(REPEAT)
38. Spinal anaesthesia is given at which level?
A. L1-2
B. L2-4
C. S1
D. Thoracic segment
Ans: B L2-4
Reference: Procedureconsult--elsevier
Spinal anesthetics have their effects at the spinal cord, which originates at the
foramen magnum of the skull and the brainstem and extends caudally to the
conus medullaris. The distal termination varies from about the level of the 3rd
lumbar vertebrae (L3) in infants to the lower border of L1 in adults.

The spinal cord is surrounded by three membranes (from central to peripheral):


the pia mater, arachnoid mater, and dura mater. It is believed that the arachnoid
mater is responsible for up to 90% of the resistance to drug migration in and out
of the CSF.

Inside the subarachnoid space are the CSF, spinal nerves, a network of
trabeculae between the two membranes, and blood vessels supplying the spinal
cord. Although the spinal cord ends at about L1 in adults, the subarachnoid space
continues to about the second sacral vertebrae (S2).
Posterior to the epidural space is the ligamentum flavum, which extends from the
foramen magnum to the sacral hiatus. Immediately posterior to the ligamentum
flavum are the lamina and spinous processes of the vertebral bodies or the
interspinous ligaments. Posterior to these structures is the supraspinous
ligament, which joins the vertebral spines.

Anatomic landmarks most important to performance of spinal anesthesia are the


iliac crests, the midline of the back, and the vertebral spinous processes.
Palpation of the midline of the back identifies the spinous processes and vertebral
interspaces in most patients but may be difficult in obese patients. A line drawn
between the upper borders of the iliac crests across the midline of the back
identifies the approximate level of L4 or the L4-L5 interspace.

Spinal anesthesia is usually performed at the level of the L3 or L4


vertebrae in the adult patient, because the spinal needle is introduced
below the level at which the spinal cord ends.

39. A child presented with mild fever little breathlessness was treated and she
improved over 4 days but later deteriorated again with fever and more
breathlessness. x ray showed hyperlucency. diagnosis?
A.bronchiolitis obliterans
B.alveolar proteinosois
C.bronchitis D. Asthma
Ans: A Bronchiolitis obliterans(REPEAT)
40. Which of the following passes through foramen magnum?
A. Internal Carotid Artery
B. Sympathetic chain
C. Hypoglossal Nerve
D. Vertebral Artery
Ans: D Vertebral artery (REPEAT)
41. McKeon's theory on reduced prevalence of TB?
A. Increased awareness and knowledge
B. Medical advancement answer
C. Behavioural modification
D. Social and environmental factor
Ans: D Social and environmental factors
Reference:” McKeown and the Idea That Social Conditions Are Fundamental
Causes of Disease Bruce G. Link, PhD and Jo C. Phelan, PhD “

THE MCKEOWN THESIS STATES that the enormous increase in population


and dramatic improvements in health that humans have experienced
over the past 2 centuries owe more to changes in broad economic and
social conditions than to specific medical advances or public health
initiatives. The thesis gives center stage to social conditions as root causes of
the health of populations. On the basis of new data and numerous revisitations,
however, Colgrove2 tells us that the thesis has been “overturned” and the theory
“discredited.” Whither, then, the idea that social conditions require prominence in
any complete understanding of the health of populations? When we turn away
from “the thesis,” do we accept an “antithesis” asserting that the role of social
conditions is insignificant?
42. Which among the following is not a cause of fasting hypoglycemia?
A. Glucagon excess
B. Glucose 6 phospatase deficiency
C. Ureamia
D. Glycogen synthase deficiency
Ans: A Glucagon excess
Reference:Harrison Principles of Internal Medicine 17th ed
Glucose 6 phosphatase defeciency leads to inability to mobilize glucose and hence
can cause
Glycogen synthase def leads to glycogen deficiency and hence during fasting
patient goes hypo
In uremia liver metabolism is hampered and hence hypo is possible
In glucagon excess there is HYPERglycemia and hence the answer.
This is the table from Harrison 17th ed :

Fasting (Postabsorptive) Hypoglycemia

Drugs
Especially insulin, sulfonylureas, ethanol
Sometimes quinine, pentamidine
Rarely salicylates, sulfonamides, others
Critical illnesses
Hepatic, renal, or cardiac failure
Sepsis
Inanition
Hormone deficiencies
Cortisol, growth hormone, or both
Glucagon and epinephrine (in insulin-deficient diabetes)
Non-beta-cell tumors
Endogenous hyperinsulinism
Insulinoma
Other beta cell disorders
Insulin secretagogue (sulfonylurea, other)
Autoimmune (autoantibodies to insulin or the insulin receptor)
Ectopic insulin secretion
Disorders of infancy or childhood
Transient intolerance of fasting
Congenital hyperinsulinism
Inherited enzyme deficiencies

43. Mineralocorticoid receptor is not present in?


A. Liver
B. Colon
C. Hippocampus
D. Kidney
Ans:A Liver (REPEAT)
44. Prolonged treatment with INH leads to deficiency of?
A. Pyridoxine
B. Thiamine
C. Pantothenic acid
D. Niacin
Ans: A Pyridoxine
Reference: Harrison 17th ed table of vitamins
Vitamin Defeciency symptoms: Dietary Level per Day Factors
B6 Seborrhea, glossitis Associated with Overt contributing to
convulsions, neuropathy, Deficiency in Adults: deficiency:
depression, confusion, <0.2 mg Alcoholism,
microcytic anemia isoniazid

45. Which is the most reliable objective sign of identifying pulmonary plethora in
chest X-ray?
A. Diameter of the main pulmonay artery >16mm
B. Diameter of the lt pulmonay artery >16mm
C. Diameter of the decending Rt pulmonay artery >16mm
D. Diameter of the decending Lt pulmonay artery >16mm
Ans: C Diameter of the decending Rt pulmonay artery >16mm (REPEAT)
46. Necrotizing lymphadenitis is seen in?
A. Kimura disease
B. Kikuchi disease
C. Hodgkin disease
D. Castelman disease
Ans: B Kikuchi disease (a type of Modified REPEAT)
Reference: Harrison 17th ed and eMEDICINE
Kikuchi disease, also called histiocytic necrotizing lymphadenitis or Kikuchi-Fujimoto
disease
The most common clinical manifestation of Kikuchi disease is cervical lymphadenopathy
Kimura disease is a chronic inflammatory disorder of unknown etiology that most commonly
presents as painless, unilateral cervical lymphadenopathy or subcutaneous masses in the
head or neck region. The disorder received its current name in 1948, when Kimura et
al2 noted the vascular component and referred to it as an "unusual granulation combined with
hyperplastic changes in lymphoid tissue."
Lymph-Node Tuberculosis (Tuberculous Lymphadenitis) from Harrison
17th ed:
The most common presentation of extrapulmonary tuberculosis (>40% of cases
in the United States in recent series), lymph-node disease is particularly frequent
among HIV-infected patients. In the United States, children and women
(particularly non-Caucasians) also seem to be especially susceptible. Once caused
mainly by M. bovis, tuberculous lymphadenitis is today due largely to M.
tuberculosis. Lymph-node tuberculosis presents as painless swelling of the lymph
nodes, most commonly at posterior cervical and supraclavicular sites (a condition
historically referred to as scrofula). Lymph nodes are usually discrete and
nontender in early disease but may be inflamed and have a fistulous tract
draining caseous material. Associated pulmonary disease is seen in >40% of
cases. Systemic symptoms are usually limited to HIV-infected patients. The
diagnosis is established only by fine-needle aspiration or surgical biopsy. AFB are
seen in up to 50% of cases, cultures are positive in 70–80%, and histologic
examination shows granulomatous lesions. Among HIV-infected patients,
granulomas usually are not seen. Differential diagnosis includes a variety of
infectious conditions, neoplastic diseases such as lymphomas or metastatic
carcinomas, and rare disorders like Kikuchi disease (necrotizing histiocytic
lymphadenitis), Kimura's disease, and Castleman's disease.

47. NARP syndrome is seen in?


A. Mitochondrial function disorder
B. Glycogen storage disorder
C. Lysosomal storage disorder
D. Lipid storage disorder
Ans: A Mitochondrial disorder(REPEAT)
48. A 65 yrs old lady presented with a swollen and painful knee. On examination,
she was found to have grade III osteoarthritic changes. What is the “BEST
COURSE OF ACTION”?
A. Conservative management
B. Arthroscopic washing
C. Partial knee replacement
D. Total knee replacement
Ans: D Total knee replacement
49. Causes of primary amenorrhoea are all except?
A. Rokintasky syndrome
B. Kallaman syndrome
C. Sheehan syndrome D.Turner syndrome
Ans: C Sheehan syndrome (REPEAT)
50. Integrase inhibitor approved for treatment of HIV is?
A. Raltegravir
B. Indinavir
C. Lopinavir
D. Elvitegravir
Ans: A Raltegravir
Reference: Pasting a net reference since the texts don’t have the drug....anyway
Raltegravir and maraviroc are given in Sparsh Pharmac
Raltegravir (MK-0518, brand name Isentress) is an antiretroviral drug produced by Merck &
Co., used to treat HIV infection.[1] It received approval by theU.S. Food and Drug
Administration (FDA) in October 2007, the first of a new class of HIV drugs, the integrase
inhibitors, to receive such approval.
SOME DOCTORS ARE OF THE OPINION THAT RALEGRASE WAS THERE IN
OPTIONS..AND FEEL ELVITEGRAVIR SHOULD BE THE ANSWER.BUT ELVITEGRAVIR
IS STILL NOT AN APPROVED DRUG OF THE SAME CLASS.
51. Deoxygenated blood is not seen in
a. Pulmonary artery
b. Umbilical artery
c. Umbilical vein
d. Renal vein
Ans: C Umbilical vein
52. All of the following are pneumatic bones except?
A. Frontal
B. Ethmoid
C. Mandible
D. Maxilla
Ans: C Mandible
53. Which of the following is not a contraindication for pregnancy?
A. WPW syndrome
B. Pulmonary hypertension
C. Eisenmenger syndrome
D. Marfan syndrome with aortic root dilatation
Ans: A WPW syndrome(REPEAT)
54. Which of the following antihypertensive drugs is contraindicated in a patient
on Lithium in order to prevent toxicity?
A. Clonidine
B. Beta blockers
C. Calcium channel blockers
D. Diuretics
Ans: D Diuretics
Reference: Goodman and gilman Pharmacology
Most of the renal tubular reabsorption of Li+ occurs in the proximal tubule.
Nevertheless, Li+ retention can be increased by any diuretic that leads to
depletion of Na+, particularly the thiazides .
Renal excretion can be increased by administration of osmotic diuretics,
acetazolamide, or aminophylline, although they are of little help in the
management of Li+ intoxication. Triamterene may increase excretion of Li+,
suggesting that some reabsorption of the ion may occur in the distal nephron.
However, spironolactone does not increase the excretion of Li+.
55. Superior vena caval syndrome is most commonly caused by?
A. Lymphoma
B. Small cell lung ca
C. Non small cell lung ca
D. Secondary tumours
Ans: B Small cell carcinoma(REPEAT)
Reference: Schwartz surgery and Harrison 17th ed
56. Which of the following is not an adverse effect of thalidomide?
A. Diarrhoea
B. Teratogenicity
C. DVT
D. Peripheral neuropathy
Ans: A Diarrhea
Reference: Goodman and Gilman Pharmacology
The most common adverse effects reported in cancer patients are sedation
and constipation, while the most serious one is treatment-emergent
peripheral sensory neuropathy, which occurs in 10% to 30% of patients with
MM or other malignancies in a dose- and time-dependent manner. Thalidomide-
related neuropathy is an asymmetric, painful, peripheral paresthesia with
sensory loss, commonly presenting with numbness of toes and feet, muscle
cramps, weakness, signs of pyramidal tract involvement, and carpal tunnel
syndrome. The incidence of peripheral neuropathy increases with higher
cumulative doses of thalidomide, especially in elderly patients. Although clinical
improvement typically occurs upon prompt drug discontinuation, long-standing
residual sensory loss can occur. Particular caution should be applied in cancer
patients with preexisting neuropathy (e.g., related to diabetes) or prior exposure
to drugs that can cause peripheral neuropathy (e.g., vinca alkaloids or
bortezomib), especially since there has been little progress in defining effective
strategies to alleviate neuropathic symptoms. An increasing incidence of
thromboembolic events in thalidomide-treated patients has been
reported, but mostly in the context of thalidomide combinations with other
drugs, including steroids and particularly anthracycline-based chemotherapy
(Zangari et al., 2001), and with very low incidence with single-agent thalidomide
treatment.
57. Blount’s disease is:
A. Genu valgum
B. Genu varum
C. Genu recurvatum
D. Menisceal injury
Ans: B Genu varum
Reference:eMEDICINE(don’t have the orthobook with me right now)
Blount disease is an uncommon growth disorder characterized by disordered
ossification of the medial aspect of the proximal tibial physis, epiphysis,
and metaphysis. This progressive deformity is manifested by varus angulation
and internal rotation of the tibia in the proximal metaphyseal region
immediately below the knee
58. A teenaged girl complains of pain in knee on climbing stairs and on getting up
after sitting for a long time. What is the probable diagnosis?
A. Chondromalacia patellae
B. Plica syndrome
C. Bipartite patella
D. Patello-femoral osteoarthritis
Ans: A Chondromalacia patellae
59. Which of the following is not included in parenteral nutrition?
A. Fat
B. Carbohydrate
C. Fibre
D. Micronutrients
Ans: C Fibre
60. Sparrow marks are seen in?
A. Gunshot injuries
B. Stab injury of face
C. Vitriolage
D. Windshield glass injury
Ans: D Winshield glass injury
Reference: Forensic pathology by David.J.Williams
“The occupants of the first row seats in a motor vehicle accident may
demonstrate facial injuries due to contact with windshield glass called
sparrow’s foot.”

61. Amphotericin B causes deficiency of?


A. Na
B. Ca
C. K
D. Mg
Ans: C Potassium
Reference: Goodman and Gilman Pharmacology
Renal tubular acidosis and renal wasting of K+ and Mg2+ also may be seen
during and for several weeks after therapy. Supplemental K+ is required in
one-third of patients on prolonged therapy.
62. All are seen in injury to common peroneal nerve except?
A. Loss of sensation over sole
B. Foot drop
C. Injury to neck of fibula
D. Loss of dorsiflexion of toe
Ans: A loss of sensation over the sole
63. Cause of premature death in schizophrenia?
A. Homicide
B. Suicide
C. Toxicity of antipsychotic drug
D. Hospital acquired infection
Ans:B Suicide
Reference:Kaplan and sadock’s Synopsis Of Psychiatry(A STAIGHT LINE TO LINE
FROM THE TEXT)
Suicide

Suicide is the single leading cause of premature death among people


with schizophrenia. Suicide attempts are made by 20 to 50 percent of the
patients, with long-term rates of suicide estimated to be 10 to 13 percent. These
numbers reflect an approximately 20-fold increase over the suicide rate in the
general population. Often, suicide in schizophrenia seems to occur out of the blue
without prior warnings or expressions of verbal intent. The most important factor
is the presence of a major depressive episode. Epidemiological studies indicate
that up to 80 percent of schizophrenia patients may have a major depressive
episode at some time in their lives. Some data suggest that those patients with
the best prognosis (few negative symptoms, preservation of capacity to
experience affects, better abstract thinking) can paradoxically also be at highest
risk for suicide. The profile of the patient at greatest risk is a young man
64. Epileptic potential is present in
A. Desflurane
B. Halothane
C. Sevoflurane
D. Ether
Ans: C Sevoflurane
65. Which of the following anesthetic drugs is contraindicated in a patient with
hypertension?
A. Ketamine
B. Propofol
C. Etomidate
D. Diazepam
Ans:A Ketamine
66. All of the following decrease bone resorption in osteoporosis except?
A. Alendronate
B. Etidronate
C. Strontium
D. Teriparatide
Ans: D Teripartide
Reference: Goodman and Gilman Pharmacology
Parathyroid Hormone (PTH)
Continuous administration of PTH or high circulating PTH levels achieved in
primary hyperparathyroidism causes bone demineralization and osteopenia.
However, intermittent PTH administration promotes bone growth. Selye first
described the anabolic action of PTH some 80 years ago, but this observation was
largely ignored and generally forgotten. Beginning in the 1970s, studies focused
on the anabolic action of PTH, culminating with FDA approval of synthetic human
34-amino-acid amino-terminal PTH fragment [hPTH(1–34), teriparatide] for use
in treating severe osteoporosis (Hodsman et al., 2005). Full-length PTH(1–84) is
likely to be approved in the near future; its benefits over PTH(1–34) are unclear.
Absorption, Fate, and Excretion
Pharmacokinetics and systemic actions of teriparatide on mineral metabolism are
the same as for PTH. Teriparatide is administered by once-daily subcutaneous
injection of 20 g into the thigh or abdomen. With this regimen, serum PTH

concentrations peak at 30 minutes after the injection and decline to undetectable


concentrations within 3 hours, whereas the serum calcium concentration peaks at
4 to 6 hours after administration. Based on aggregate data from different dosing
regimens, teriparatide bioavailability averages 95%. Teriparatide clearance
averages 62 L/hour in women and 94 L/hour in men, which exceeds normal liver
plasma flow, consistent with both hepatic and extrahepatic PTH removal. The
serum half-life of teriparatide is approximately 1 hour when administered
subcutaneously versus 5 minutes when administered intravenously. The longer
half-life following subcutaneous administration reflects the time required for
absorption from the injection site. The elimination of PTH(1–34) and full-length
PTH proceeds by nonspecific enzymatic mechanisms in the liver, followed by renal
excretion.
Clinical Effects
In postmenopausal women with osteoporosis, teriparatide increases BMD and
reduces the risk of vertebral and nonvertebral fractures. Several laboratories
have examined the effects of intermittent PTH on BMD in patients with
osteoporosis. In these studies, teriparatide increased axial bone mineral, although
initial reports of effects on cortical bone were disappointing. Coadministration of
hPTH(1–34) with estrogen or synthetic androgen led to impressive gains in
vertebral bone mass or trabecular bone. However, in some early studies there
was only maintenance or even loss of cortical bone. Vitamin D insufficiency in
patients at baseline or pharmacokinetic differences involving bioavailability or
circulating half-life may have contributed to observed differences on cortical
bone. The most comprehensive studies to date established the value of daily
hPTH(1–34) administration on total BMD, with significant elevations of BMD in
lumbar spine and femoral neck and with significant reductions of vertebral and
nonvertebral fracture risk in osteoporotic women (Neer et al., 2001) and men
(Finkelstein et al., 2003).
Candidates for teriparatide treatment include women who have a history of
osteoporotic fracture, who have multiple risk factors for fracture, or who failed or
are intolerant of previous osteoporosis therapy.
Adverse Effects
In rats, teriparatide increased the incidence of bone tumors, including
osteosarcoma (Vahle et al., 2004). The clinical relevance of this finding is unclear,
especially since patients with primary hyperparathyroidism have considerably
higher elevations of serum PTH without a greater incidence of osteosarcoma.
Nonetheless, teriparatide should not be used in patients who are at increased
baseline risk for osteosarcoma (including those with Paget's disease of bone,
unexplained elevations of alkaline phosphatase, open epiphyses, or prior radiation
therapy involving the skeleton). Full-length PTH(1–84), which is in clinical trials,
has not been associated with osteosarcomas. Other adverse effects have included
exacerbation of nephrolithiasis and elevation of serum uric acid levels.
67.Ondoni cells and Haller cells are associated with the following structures
respectively?
A. Optic nerve and Orbital floor answer
B. Optic nerve and Internal carotid artery
C. Internal carotid artery and Optic nerve
D. Orbital floor and Internal carotid artery
Ans: Optic nerve and Orbital floor(REPEAT)

68. Pain sensation from the ethmoid sinus is carried by :


A. Frontal nerve
B. Lacrimal nerve
C. Nasociliary nerve
D. Infraorbital nerve
Ans: C Nasociliary nerve which divides into the ethmoidal branches to supply
the ethmoidal sinus

69. 28-Which among the following is not used to treat alcohol dependence?
A. Flumazenil
B. Acamprosate
C. Naltrexone
D. Disulfiram
Ans: A Flumazenil(a MODIFIED REPEAT)

70. A 40yr old patient has a single kidney with an exophytic mass of 4 cm size at it’s
lower pole. Which among the following is the best course of action?
A. Partial nephrectomy
B. Radical nephrectomy with dialysis
C. Radical nephrectomy with immediate renal transwerplant
D. Observation

Ans: A Partial nephrectomy since it is the preferred for tumors less than or equal to
4cm and at poles.(OPEN FOR SCRUTINY)

71. 29-Which among the following is the most common fungal infection seen in immuno
competent patients?
A. Aspergillus
B. Candida
C. Cryptococcus
D. Mucor

Ans: A Aspergillus Better reference awaited


Reference: Harrison 17th ed
The required size of the infecting inoculum is uncertain; however, only intense exposures
(e.g., during construction work, handling of moldy bark or hay, or composting) are
sufficient to cause disease in healthy immunocompetent individuals.
An increasing incidence of invasive aspergillosis in medical intensive care units suggests
that, in patients who are not immunocompromised, temporary abrogation of protective
responses as a result of glucocorticoid use or a general anti-inflammatory state is a
significant risk factor.

72. All are seen in Argyl Robertson pupil except?


A. Near reflex normal
B. Direct reflex absent
C. Consensual reflex normal
D. Vision normal

Ans: C Consensual reflex is normal

Argyll Robertson pupil


This is caused by neurosyphilis and is characterised by the
following:
• Involvement is usually bilateral but asymmetrical
• The pupils arc small and irregular.
• Light-near dissociation. • The pupils are very difficult to dilate.

73. 31-A 5 year old boy while having dinner suddenly becomes aphonic and is brought to
the casulty for the complaint of respiratory distress. What should be the appropriate
management?
A. Cricothyroidotomy
B. Emergency tracheostomy
C. Humidified oxygen
D. Heimlich maneuver

Ans: D Heimlich manoeveure(OPEN TO SCRUTINY TILL A VERY SATISFYING ANSWER


IS GOT)

This is the AHA guidelines for management of any conscious choking patient....The
question in the exam had a conscious,aphonic patient in respiratory distress...Now how
to manage.....Most of the discussions in the forum give other answer feeling that
Heimlich has to be performed at the site only...lets C the guidelines and find the
answer...

1. Recognise signs of choking


2. Call for help
3. Use Heimlich manoeveure until the patient recovers OR LOSES
CONSCIOSNESS
4. Place the victim in a supine position ..open the mouth and perform a finger
sweep
5. Open the airway and attempt to ventilate
6. If unsuccessful give upto 5 heimlich manoeveures
7. Repeat 5 and 6
8. If expert has arrived he has to perform a laryngoscopy and try to remove the
foreign body if not already removed
9. FAILING ALL ATTEMPTS ONLY CRICOTHYOTOMY SHOULD BE PERFORMED.

74.Which among the following is a branch from the trunk of brachial plexus?
A. Subscapular nerve
B. Long thoracic nerve
C. Anterior thoracic nerve
D. Nerve to subclavius

Ans:D Nerve to subclavius....If suprascapular nerve was there in the options then
both the answers are correct.

75.Orthotolidine test is used for detecting:


A. Chlorine
B. Nitrites
C. Nitrates
D. Ammonia
Ans: A Chlorine
76. Which among the following is the most common tumour associated with
neurofibromatosis in a child?
A. Juvenile myelomonocytic leukemia
B.Acute lymphoblastic leukemia
C. Acute monocytic leukemia
D. Acute myeloid leukemia

Ans: Juvenile myelomonocytic leukemia


Reference: Nelson Textbook Of Pediatrics 18th ed
Juvenile Chronic Myelogenous Leukemia
Juvenile chronic myelogenous leukemia (JCML), also known as juvenile myelomonocytic
leukemia, is a clonal proliferation of hematopoietic stem cells that typically affects
children <2 yr of age. Patients with this disease do not have the Philadelphia
chromosome that is characteristic of CML. Patients with JCML present with rashes,
lymphadenopathy, and splenomegaly. Analysis of the peripheral blood often shows an
elevated leukocyte count and also may show thrombocytopenia and the presence of
erythroblasts. The bone marrow shows a myelodysplastic pattern, with blasts accounting
for <30% of cells. No distinctive cytogenetic abnormalities are seen. JCML is rare,
constituting <2% of all cases of childhood leukemia. Therapeutic reports are largely
anecdotal. Patients with neurofibromatosis type 1 have a predilection for this type of
leukemia. Stem cell transplantation offers the best opportunity for cure, but much less
so than for classic CML.

77. Diaphragm develops from all except:


A. Septum transwerversum
B. Dorsal mesocardium
C. Pleuroperitoneal membrane
D. Cervical myotomes

Ans: B Dorsal mesocardium

78. Erythema nodosum is seen in all except:


A. Pregnancy
B. Tuberculosis
C. SLE
D. Chronic pancreatitis
Ans: D Chronic pancreatitis
Reference:eMEDICINE
Causes
Currently, the most common cause of erythema nodosum is streptococcal infection in children and
streptococcal infection and sarcoidosis in adults.3 Numerous other causes have been reported.4 The
causes reported most often in the literature are as follows:
• Bacterial infections: Streptococcal infections are one of the most common causes of erythema
nodosum. Tuberculosis was an important cause in the past, but it has decreased
dramatically as a cause for erythema nodosum; however, it still must be excluded, especially
in developing countries.5,6 Yersinia enterocolitica is a gram-negative bacillus that causes acute
diarrhea and abdominal pain; it is a common cause of erythema nodosum in France and
Finland.7,8,9 Mycoplasma pneumoniae infection may cause erythema nodosum. Erythema
nodosum leprosum clinically resembles erythema nodosum, but the histologic picture is that
of leukocytoclastic vasculitis. Lymphogranuloma venereum may cause erythema
nodosum. Salmonella infection may cause erythema nodosum. Campylobacter infection may
cause erythema nodosum.
• Fungal infections: Coccidioidomycosis (San Joaquin Valley fever) is the most common cause
of erythema nodosum in the American Southwest. In approximately 4% of males and 10% of
females, the primary fungal infection (which may be asymptomatic or involve symptoms of
upper respiratory infection) is followed by the development of erythema nodosum. Lesions
appear 3 days to 3 weeks after the end of the fever caused by the fungal infection.
Histoplasmosis may cause erythema nodosum. Blastomycosis may cause erythema
nodosum.
• Drugs: Sulfonamides and halide agents are an important cause of erythema nodosum. Drugs
more recently described to cause erythema nodosum include gold and sulfonylureas. Oral
contraceptive pills are implicated in an increasing number of reports.
• Enteropathies: Ulcerative colitis and Crohn disease may trigger erythema nodosum.
Erythema nodosum associated with enteropathies correlates with flares of the disease. The
mean duration of chronic ulcerative colitis before the onset of erythema nodosum is 5 years,
and erythema nodosum is controlled with adequate therapy of the colitis. Erythema nodosum
is the most frequent dermatologic symptom ininflammatory bowel diseases, and it is strongly
associated with Crohn disease.10
• Hodgkin disease and lymphoma: Erythema nodosum associated with non-Hodgkin lymphoma
may precede the diagnosis of lymphoma by months. Reports of erythema nodosum preceding
the onset of acute myelogenous leukemia have been published.11
• Sarcoidosis: The most common cutaneous manifestation of sarcoidosis is erythema
nodosum. A characteristic form of acute sarcoidosis involves the association of erythema
nodosum, hilar lymphadenopathy, fever, arthritis, and uveitis, which has been termed Löfgren
syndrome. This presentation has a good prognosis, with complete resolution within several
months in most patients. HLA-DRB1*03 is associated with Löfgren syndrome. Most DRB1*03-
positive patients have resolution of their symptoms within 2 years; however, nearly half of
DRB1*03-negative patients have an unremitting course.12
• Behçet disease (associated with erythema nodosum)
• Pregnancy: Some patients develop erythema nodosum during pregnancy, most frequently
during the second trimester. Repeated episodes occur with subsequent pregnancies or with
the use of oral contraceptives.

79. Posterior relations of head of pancreas are all except?


A. Common bile duct
B. First part of duodenum
C. Aorta
D. Inferior vena cava
Ans: B First part of Duodenum
80. Multiple sebaceous cysts seen in:
A. Gardner’s syndrome
B. Turcot syndrome
C. Muir Torre syndrome
D. Cowden syndrome
Ans:A Gerdener syndrome
If u believe the option was Sebaceous adenoma then the answer is Muir Torre

81. Which among the following is the hallmark of acute inflammation?


A. Vasoconstriction
B. Stasis
C. Vasodilation and increase in permeability
D. Leukocyte margination
Ans: C Vasodilatation and increased vascular permeability(REPEAT)
82.True about epidural opioids are all except?
A. Acts on dorsal horn substantia gelatinosa
B. Can cause Itching
C. Function of the intestines are not affected
D. Can cause respiratory depression

Ans: C Function of the intestine is not affected (better among the options)

Reference: Pharmacology By KATZUNG(THE QUESTION HAS BEEN TAKEN LINE TO LINE


FROM TEXT---READ BELOW)

Because of their direct action on the superficial neurons of the spinal


cord dorsal horn, opioids can also be used as regional analgesics by administration into
the epidural or subarachnoid spaces of the spinal column. A number of studies have
demonstrated that long-lasting analgesia with minimal adverse effects can be achieved
by epidural administration of 3–5 mg of morphine, followed by slow infusion through a
catheter placed in the epidural space. It was initially assumed that the epidural
application of opioids might selectively produce analgesia without impairment of motor,
autonomic, or sensory functions other than pain. However, respiratory depression can
occur after the drug is injected into the epidural space and may require reversal
with naloxone. Effects such as pruritus and nausea and vomiting are common
after epidural and subarachnoid administration of opioids and may also be
reversed with naloxone if necessary. Currently, the epidural route is favored
because adverse effects are less common.

83. All are true about pheochromocytoma except?


A. 90% are malignant
B. 95% occur in the abdomen
C. They secrete catecholamines
D. They arise from sympathetic ganglions

Ans: A 90% are malignant...as per the rule of 10...only 10% are malignant
Reference:Harrison 17th ed
Epidemiology
Pheochromocytoma is estimated to occur in 2–8 out of 1 million persons per year, and
about 0.1% of hypertensive patients harbor a pheochromocytoma. Autopsy series reveal
prevalence figures of 0.2%. The mean age at diagnosis is about 40 years, although the
tumors can occur from early childhood until late in life. The "rule of tens" for
pheochromocytomas states that about 10% are bilateral, 10% are extraadrenal, and
10% are malignant. However, these percentages are higher in the inherited
syndromes.

84. Campylobacter jejuni false is:

a.commonest cause of campylobacteriosis


b.polutry source of infection
c.humans are reservoir
d.associated with GBS

Answer:C humans are the only reservoir[REPEAT AIIMS09]

85. Which of the following does not cause indoor air pollution?
A. CO
B. Nitrogen dioxide
C. Radon
D. Mercury vapour
Ans:D Mercury vapour

86. Most important and potential agent that can be used in bioterrorism:
A. Plague
B. Small pox
C. TB
D. Clostridium botulinum

Ans:B small pox


87. Earliest to be diagnosed by antenatal USG is?
A. Anencephaly
B. Prosencephaly
C. Meningocele
D. Spina bifida
Ans:A Anencephaly(REPEAT)

88. 151. Dose of radiation required for development of haematological syndrome is?
A. 2.5-5 cGy
B. 10 cGy
C. 100 cGy
D. 200 cGy

Ans:C 100cgy
Seivert/s=gray/s
1 gray=100cgy=10000rad

89. Intraoperative myocardial infarction is best diagnosed by:


A. ECG ??
B. Invasive arterial pressure
C. Central venous pressure
D. Transwer esophageal echo

Ans: D TEE(REPEAT)

90. Pseudoisomorphic phenomenon seen in


A. Psoriasis
B. Lichen planus
C. Vitiligo
D. Plane warts
Ans: D Plane warts
pseudoisomorhic phenomenon "

"Boyd and Neldner have classified all reported cases of Koebner phenomenon into four
different groups:

1. True isomorphic phenomenon: There appear to be three disease processes that


display the true isomorphic response of Koebner: psoriasis, lichen planus and vitiligo;
2. Pseudoisomorphic phenomenon: The Koebner phenomenon seen in infectious
diseases, e.g. warts, molluscum contagiosum and pyoderma gangrenosum;

another reference ..

"ROXBURGS" 17TH EDITION

it says .. on pg 21 & 130 ... the conditions causing isomorphic phenomenon ...
1) psoriasis
2)lichen planus
3)DLE

another is from wikipedia ....


but he has complicated the things .. yet in the last sentence he says the truth abt the
plane warts .. see this too ..

"Koebner phenomenonFrom Wikipedia, the free encyclopedia Heinrich Köbner (1838-


1904)

The Koebner phenomenon, also called the "Koebner response" or the "isomorphic
response", refers to skin lesions appearing on lines of trauma.[1]

The Koebner phenomenon may result from either a linear exposure or irritation.
Conditions demonstrating linear lesions after a linear exposure to a causative agent
include: molluscum contagiosum, warts and toxicodendron dermatitis (a dermatitis
caused by a genus of plants including poison ivy).

Warts and molluscum contagiosum lesions can be spread in linear patterns by self-
scratching ("auto-inoculation"). Toxicodendron dermatitis lesions are often linear from
brushing up against the plant.

Causes of the Koebner phenomenon that are secondary to scratching rather than an
infective or chemical cause include vitiligo, psoriasis, lichen planus, lichen nitidus,
pityriasis rubra pilaris, and keratosis follicularis (Darier disease).

The Koebner phenomenon describes skin lesions which appear at the site of injury. It is
seen in:[2]
Psoriasis
Pityriasis rubra pilaris
Lichen planus
Lichen nitidus
Vitiligo
Lichen sclerosus
Elastosis perforans serpiginosa
warts

91. A fire breaks out during laser vocal cord surgery. What is not to be done?
A. Pouring sterile water into the oral cavity
B. Removing endotracheal tube
C. 100% oxygen after discontinuing anesthetic gases
D. Treatment with steroid & antibiotic
Ans: C 100% oxygen after discontinuing the anesthetic gases--QQQQQQQQQQQQQQQ
Reference: Could not get a Text reference....a journal paper on Management of Airway
fire during Microlaryngeal surgery provides some light
Managing fire

Remove source of fire and extinguish with water


Stop ventilation,” turn off O2 “
Mask ventilate with air, “then 100% O2 once fire is extinguished”
Laryngoscopy and rigid bronchoscopy to remove debris
Lavage and fibreoptic bronchoscopy if indicated by airway injury
Common pattern is worst injury at the surgical site and little distal injury
If severe injury
“Maintain ventilation”
Consider low tracheostomy
“IV corticosteroids may be helpful”
CXR, ABG with co-oximetry for smoke inhalation assessment

92. Ovoalbumin was injected into a rabbit. What antibody will it produce initially?
A. IgG
B. IgM
C. IgE
D. IgD
Ans: B IgM

93. Which insect among the following is not resistant to DDT?


A. Musca domestica
B. Phlebotomus
C. Culex
D. Anopheles stephensi
Ans:B Phlebotomus

94. Which virus among the following is least likely to cross placenta?
A. Rubella
B. Herpes simplex
C. HIV
D. HBV

Ans:D HBV
95. About yaws all are true except:
A. Caused by Treponema pertenue
B. Transwermitted non-venerally
C. Secondary yaws can involve bones
D. Last stages involve heart and nerves

Ans: D Late stages involve heart and nerves

Reference:Harrison 17th ed

Yaws
Also known as pian, framboesia, or bouba, yaws is caused by T. pallidum subspecies
pertenue and is characterized by the development of one or several primary lesions
("mother yaw"), which is followed by the appearance of multiple disseminated skin
lesions. All early skin lesions are infectious and may persist for many months; cutaneous
relapses are common during the first 5 years. Late manifestations, affecting 10% of
untreated persons, are destructive and can involve skin, bone, and joints.
The infection is transmitted by direct contact with infectious lesions, often during play or
group sleeping, and may be enhanced by disruption of the skin by insect bites or
abrasions. After an average of 3–4 weeks, the first lesion begins as a papule—usually on
an extremity—and then enlarges (particularly during moist warm weather) to become
papillomatous or "raspberry-like" (thus the name "framboesia") (Fig. 163-2). Regional
lymphadenopathy develops, and the lesion usually heals within 6 months; dissemination
is thought to occur during the early weeks of infection. A generalized secondary
eruption, accompanied by generalized lymphadenopathy, appears either concurrent with
or following the primary lesion, may take several forms (macular, papular, or
papillomatous), and may become secondarily infected with other bacteria. Painful
papillomatous lesions on the soles of the feet result in a painful crablike gait ("crab
yaws"), and periostitis may result in nocturnal bone pain and polydactylitis.

Late yaws is manifested by gummas of the skin and long bone, hyperkeratoses
of the palms and soles, osteitis and periostitis, and hydrarthrosis. The late
gummatous lesions are characteristically extensive. Destruction of the nose, maxilla,
palate, and pharynx is termed gangosa and is similar to the destructive lesions seen in
leprosy and leishmaniasis.
96. Weight gain in pregnancy is related to all except?
A. Ethnicity
B. Smoking
C. Socioeconomic status
D. Pre conceptional weight
Ans: B Smoking (REPEAT)

97. A 3.8 kg baby of a diabetic mother developed seizures 16 hours after birth. Most
probable cause is?
A. Hypoglycemia
B. Hypocalcemia
C. Birth asphyxia
D. Intra ventricular hemorrhage
Ans: A Hypoglycemia(REPEAT)

98. Regarding PCOD, all are true except?


A. High LH/FSH
B. High DHEAS
C. Very high prolactin
D. Raised LH

Ans: C Very high prolactin(REPEAT)

99. Which anesthetic modality is to be avoided in sickle cell disease?


A. General anesthesia
B. Brachial plexus block
C. IV regional anesthesia
D. Spinal

Ans: C IV regional anesthesia(REPEAT)

100. Best marker for intrahepatic cholestasis of pregnancy is?


A. AST and ALT
B. Bile acid
C. Bilirubin
D. ALP

Ans: B Bile acids(REPEAT)

101. Which of the following is the most probable diagnosis in a young patient with loss of
central vision and a normal ERG with no family history?
A. Best's disease
B. Stargardt's disease
C. Retinitis pigmentosa
D. Macular hole

Ans:B Stargardt disease

Clincal ophthalmology by Vaughan and Kanski Opthalmology:

Stargardt’s Disease

The presentation is in the first to second decades with bilateral, gradual impairment of
central vision which may
be out of proportion to the macular changes. so that thechild may be suspected of
malingering.

Both Eale’s and Stargardt’s can manifest with loss of central vision and normal
ERG....but stargardt is more common and it is autosomal recessive wheras Eales is AD
and less common...

102. Renal calculi associated with proteus infection is:


A. Uric acid
B. Triple phosphate
C. Calcium oxlalate
D. Xanthine
Ans:B Triple phosphate(REPEAT)

103. The primary action of NO in git is?


A. Vasodilatation
B. Vasoconstriction
C. GI smooth muscle inhibition
D. Secretomotor

Ans:C GI smooth muscle inhibition(REPEAT)

104. A 40 year old female underwent surgery. Post operatively she told the anaesthetist
that she was aware of per-operative events. Individual intraoperative awareness is
evaluated by (to prevent such instances from occurring)?
A. Pulse oximetry
B. Colour doppler
C. Bispectral index
D. End tidal CO2

Ans:C BIS

Refernce: Quantitative EEG, event-related potentials and neurotherapy


By Juri Kropotov

Bispectral index is used to asses the depth of anesthesia.Its a good measure of brain activites in
sleep wakefulness cycle.

105. All of the following helps in generating oxygen radicals for killing bacteria within
neurophils except?
A. Superoxide dismutase
B. Nitric oxide synthase
C. Peroxidase
D. Glutathione peroxidise

Ans D. Glutathione peroxidise

Reference: Robbins pathology 8th ed

A series of enzymes acts as free radical–scavenging systems and breaks down H2O2 and
o2. These enzymes are lo-cated near the sites of generation of the oxidants and include
the following:
1. Catalase, present in peroxisomes, decomposes H2O2 (2H2O2 ➙ O2 + 2H2O).
2. Superoxide dismutases (SOD) are found in many cell types and convert O2 to
H2O2 (2O2. + 2H ➙ H2O2 + O2). This group includes both manganese–SOD
which is localized in mitochondria, and copper-zinc–SOD, which is found in the
cytosol.
3. Glutathione peroxidase also protects against injury by catalyzing free radical
breakdown (H2O2 + 2GSH ➙ GSSG [glutathione homodimer] + 2H2O, or 2OH +
2GSH ➙ GSSG + 2H2O). The intracellular ratio of oxidized glutathione (GSSG) to
reduced glutathione (GSH) is a reflection of the oxidative state of the cell and is
an important indicator of the cell's ability to detoxify ROS.
106. Most common cause of meningoencephalitis in children?
A. HSV
B. Enterovirus
C. Mumps
D. Listeria

Ans:B Enterovirus(REPEAT)

107. Principle orgenelle involved in the execution of apoptosis is?

A. Nucleus

B. Lysosome

C. Mitochondria

D. Endoplasmic reticulum

Answer:C Mitochondria(REPEAT)

108. Feature of obstructive azoospermia is?

A. High FSH, high testosterone

B. Low FSH, high testosterone

C. High FSH, low testosterone

D. Normal FSH, normal testosterone

Ans:D NORMAL Testosterone and FSH(REPEAT)

109. 'C' in C reactive protein stands for:

A. Capsular polysaccharide in pneumococcus

B. Concanavalin-a

C. Calretinin

D. Cellular

Ans:A Capsular polysaccharide of pneumococcus(REPEAT)


110. Buprenorphine is?

A. Partial agonist at Mu Receptor

B. Partial agonist at Kappa Receptor

C. Full Agonist at Mu Receptor

D. Antagonist at Kappa receptor

Ans:A Partial agonist of mu receptor

Reference: Goodman and Gilman

Buprenorphine appears to be a partial mu receptor agonist. Depending on the


dose, buprenorphine may cause symptoms of abstinence in patients who have been
receiving mu-receptor agonists for several weeks.
It antagonizes the respiratory depression produced by anesthetic doses of
fentanyl about as well as does naloxone without completely reversing opioid pain relief.
Although respiratory depression has not been a major problem, it is not clear whether
there is a ceiling for this effect (as seen with nalbuphine and pentazocine). The
respiratory depression and other effects of buprenorphine can be prevented by prior
administration of naloxone, but they are not readily reversed by high doses of naloxone
once the effects have been produced. This suggests that buprenorphine dissociates very
slowly from opioid receptors.
The half-life for dissociation from the mu receptor is 166 minutes for
buprenorphine, as opposed to 7 minutes for fentanyl Therefore, plasma levels of
buprenorphine may not parallel clinical effects. Cardiovascular and other side effects
(e.g., sedation, nausea, vomiting, dizziness, sweating, and headache) appear to be
similar to those of morphine-like opioids.
111.All are true about aprepitant except?
A. Agonist at NK1
B. Crosses blood brain barrier
C. Ameliorates nausea and vomiting of chemotherapy
D. Metabolized by CYP450

Ans:A Agonist at NK1 Receptor


Reference:Katzung Pharmacology
Neurokinin 1 (NK1) receptor antagonists have antiemetic properties that are
mediated through central blockade in the area postrema. Aprepitant is a highly
selective NK1 receptor antagonist that crosses the blood-brain barrier and occupies brain
NK1 receptors. It has no affinity for serotonin, dopamine, or corticosteroid receptors.

112. Time of occurrence of secondary haemorrhage after tonsillectomy?


A. 24 hrs
B. 6 days
C. 12 days
D. 12 hrs
Ans: 6 Days

113. A patient with head injury on examination revealed eye opening in response to
pain, inappropriate words and pain localisation. Calculate GCS?
a. 10
b. 8
c. 12
d. 14

Ans: A 10
Reference: Harrison Principles of Internal Medicine 17th ed

GLASGOW COMA SCORE

Eye Opening Verbal Verbal (Intubated) Motor Activity


(Nonintubated)
4—Spontaneous 5—Oriented and talks 5—Seems able to talk 6—Verbal
command
3—Verbal stimuli 4—Disoriented and talks 3—Questionable ability 5—Localizes to
to talk pain
2—Painful 3—Inappropriate 1—Generally 4—Withdraws to
stimuli words unresponsive pain
1—No response 2—Incomprehensible 3—Decorticate
sounds
1—No response 2—Decerebrate
1—No response

114. A primigravida in 1st trimester had sputum positive for acid fast bacillus. What is
the preferred treatment?
A. Treatment deferred till 2nd trimester
B. Category 1 DOTS
C. Category 2 DOTS
D. Category 3 DOTS
Ans:B Category 1 DOTS(REPEAT)
115. HbH is seen in?
A. Deletion of 3 alpha gene
B. Deletion of all 4 alpha genes answer
C. Deletion of 3 beta genes
D. Deletion of all 4 beta genes
Ans:A Deletion of 3 alpha genes
Reference:Harrison 17th ed
Alpha Thalassemia Syndromes:
The four classic alpha thalassemias, most common in Asians, are alpha-thalassemia-2
trait, in which one of the four alpha-globin loci is deleted; -thalassemia-1 trait, with
two deleted loci; HbH disease, with three loci deleted; and hydrops fetalis with Hb
Bart's, with all four loci deleted

116. Which among the following is a cardioprotective fatty acid?


A. Palmitic acid
B. Stearic acid
C. Oleic acid
D. Omega-3 fatty acids
Ans:D Omega-3 fatty acids(modified repeat)

117. What will you give to treat hypothyroidism in a patient with ischemic heart
disease?
A. Low dose of levothyroxine
B. Normal dose of levothyroxine
C. Do not give levothyroxine
D. Thyroid extract
Ans:A Low dose of levothyroxine(OPEN FOR DISCUSSION)

Reference---BRAUNWALD'sPRINCIPLES OF CARDIOLOGY

“SUBCLINICAL HYPOTHYROIDISM”

Subclinical hypothyroidism defined as a TSH above the upper range of the


reference population (usually >5 mIU/ml) is seen in up to 9 percent of unselected
populations, and clearly prevalence increases with advancing age. In contrast to younger
patients in whom there is a strong female predilection, in older populations, this
difference is lost. Subclinical hypothyroidism alters lipid metabolism, atherosclerosis,
cardiac contractility, and systemic vascular resistance.
Cholesterol levels rise in parallel with increments in TSH
elevations starting at 5 mIU/liter. A large study of women in Rotterdam showed that
atherosclerosis and myocardial infarction increased with odds ratios of 1.7 and
2.3 in subclinical hypothyroid women, respectively. Interestingly,
the presence of antithyroid antibodies as a measure of autoimmune thyroid disease
indicated heightened risk.
Restoration of serum TSH to normal after thyroid hormone
replacement improved lipid levels, lowered systemic vascular resistance, and
improved cardiac contractility. Patients with subclinical hypothyroidism have
prolonged isovolumic relaxation times, whereas systolic contractile function does not
change . Replacement with l-thyroxine sodium at a mean dose of 68 mg per day
(range 50 to 100 mg per day) restored isovolumic relaxation times to normal
and when compared with the same patients before therapy, systemic vascular
resistance declined and systolic function significantly improved. A variety of
studies have indicated that the changes in systemic vascular resistance result from
alterations in endothelium-dependent vasodilation.
Taken together, it seems appropriate to recommend thyroid
hormone replacement for all patients with subclinical hypothyroidism from a
cardiovascular perspective. The lack of untoward cardiac effects observed when
serum TSH levels normalize indicate that the potential benefits far outweigh
the risks of treatment.
118. Carrier state is not important in transmission of:
A. Measles
B. Typhoid
C. Polio
D. Diphtheria

Ans:A Measles(REPEAT)

119. A schizophrenic patient started on haloperidol 2 days back, comes with complaints
of torticollis and orofaciolingual movements. What is the diagnosis?
A. Acute dystonia
B. Tardive dyskinesia
C. Parkinsonism
D. Akathisia
Ans:A Acute Dystonia
Reference:Kaplan and sadock’s Synopsis of psychiatry
Neuroleptic-Induced Acute Dystonia

Diagnosis, Signs, and Symptoms

Dystonias are brief or prolonged contractions of muscles that result in obviously


abnormal movements or postures, including oculogyric crises, tongue
protrusion, trismus, torticollis, laryngeal and pharyngeal dystonias, and
dystonic postures of the limbs and trunk. Other dystonias include blepharospasm
and glossopharyngeal dystonia; the latter results in dysarthria, dysphagia, and even
difficulty in breathing, which can cause cyanosis. Children are particularly likely to
evidence opisthotonos, scoliosis, lordosis, and writhing movements. Dystonia can be
painful and frightening and often results in noncompliance with future drug treatment
regimens.

Epidemiology
The development of dystonic symptoms is characterized by their early onset during
the course of treatment with neuroleptics and their high incidence in men, in
patients younger than age 30 years, and in patients given high dosages of high-potency
medications.
Etiology
Although it is most common with intramuscular doses of high-potency antipsychotics,
dystonia can occur with any antipsychotic. The mechanism of action is thought to be
dopaminergic hyperactivity in the basal ganglia that occurs when central nervous system
(CNS) levels of the antipsychotic drug begin to fall between doses.

120. All are true about parvovirus b19 except?


A. <10 % spread by transplacental route
B. Spread by respiratory route
C. It is a DNA virus
D. Affects erythroid progenitor cells
Ans:A <10% spread by transplacental route(REPEAT)

121. Sterile pyuria is present in?


A. Renal tuberculosis
B. Chronic hydronephrosis
C. Wilm's tumour
D. Neuroblastoma
Ans:A Renal tuberculosis (REPEAT)

122. A 35 year old female has proximal weakness of muscles, ptosis and easy
fatiguability. The best test to diagnose her condition is:
A. Muscle biopsy
B. CPK
C. Edrophonium test
D. EMG

Ans:C Edrophonium test(REPEAT)

123. Denominator in Maternal Mortality Rate?


A. Total number of live births
B. Total number of married women
C. Total number of births
D. Midyear population
Ans:A Total live births(REPEAT)

124. Psammoma bodies are seen in all except?


A. Follicular carcinoma thyroid
B. Papillary carcinoma thyroid
C. Cystadenocarcinoma
D. Meningioma

Ans:A Follicular carcinoma thyroid(REPEAT)

125. Visceral larva migranswer is seen in?


A. Strongyloides
B. Ancylostoma
C. Toxocara canis
D. Visceral leishmaniasis

Ans:A Toxocara canis


126. Urethral crest is situated in:
A. Prostatic urethra
B. Membranous urethra
C. Penile urethra
D. Bulbar urethra
Ans:A Prostatic urethra

127. People were separated into relevant 5 sub groups. People were selected randomly
from these sub groups. What type of sampling was done?
A. Simple random sampling
B. Stratified Sampling
C. Cluster sampling
D. Systematic sampling

Ans:A Stratified sampling


128. All of the following are true about erlotinib except?
A. Tyrosine kinase inhibitor
B. Food decreases its absorption
C. Rashes can occur
D. Used in non small cell lung cancer when there is no response to other
chemotherapeutic agents

Ans:B Food decreases its absorption


Reference:Goodman and Gilman
Absorption, Fate, and Excretion
Erlotinib is about 60% absorbed after oral administration and its bioavailability is
substantially increased to almost 100% by food. Peak plasma levels occur 4 hours
after an oral dose. Following absorption, erlotinib is approximately 93% protein-bound to
albumin and alpha1-acid glycoprotein. Its half-life is ~36 hours. Erlotinib is metabolized
primarily by CYP3A4 and to a lesser extent by CYP1A2 and CYP1A1.
129Best marker for open nural tube defect.
A.Acetylcholinesterase
B.Pseudocholinesterase
C.AFP
D. –
Ans:A Acetylcholinesterase(REPEAT)

130. Mifepristone is used in?


A. Molar pregnancy
B. Threatened abortion
C. Fibroid
D. Ectopic pregnancy…

Ans:C Fibroid(This is one of the most fought Questions in our discussion............Shaw


is in favour of Ectopic while NOVAK-WILLIAMS-GOODMAN GILMAN are in favour of
Fibroid..So chose to pick Fibroid....To say the truth ............Not clear which to pick??)

Reference:Goodman and Gilman

Therapeutic Uses and Prospects


Mifepristone (MIFEPREX), in combination with misoprostol or other prostaglandins (see
below), is available for the termination of early pregnancy. When mifepristone is used to
produce a medical abortion, a prostaglandin is given 48 hours after the anti-progestin to
further increase myometrial contractions and ensure expulsion of the detached
blastocyst. Intramuscular sulprostone, intravaginal gemeprost, and oral misoprostol
have been used. The success rate with such regimens is >90% among women with
pregnancies of 49 days duration or less. The most severe untoward effect is vaginal
bleeding, which most often lasts from 8 to 17 days, but is only rarely (0.1% of patients)
severe enough to require blood transfusions. High percentages of women also have
experienced abdominal pain and uterine cramps, nausea, vomiting, and diarrhea due to
the prostaglandin. Women receiving chronic glucocorticoid therapy should not be given
mifepristone because of its anti-glucocorticoid activity, and the drug should be used very
cautiously in women who are anemic or receiving anticoagulants. Women over 35 years
old with cardiovascular risk factors should not be given sulprostone because of possible
heart failure (Christin-Maitre et al., 2000).
Other investigational or potential uses for mifepristone that are under development
include the induction of labor after fetal death; the induction of labor at the end of the
third trimester; treatment of endometriosis, leiomyomas, breast cancer, and
meningiomas; and as a postcoital or luteal-phase contraceptive (Spitz and Chwalisz,
2000). A major concern about long-term use is the possibility of unopposed estrogenic
effects, but this concern could be allayed by further development of selective
progesterone-receptor modulators.

131. A 55 year old man presents with history of 5 episodes of hematuria each lasting for
about 4-5 days in the past 5 years. What will be the best investigation to arrive at a
diagnosis?
A. Urine examination and microscopy
B. X-ray KUB
C. Abdominal USG
D. DTPA scan

AnsA

132. A graph of Normal blood sugar level curve and Diabetic blood sugar level curve was
shown. An area was seen overlapping towards the normal gycemic curve. A point at 120
mg/dl was shown too. Question : What does that area represent?
A. True positive
B. False positive
C. True negative
D. False negative

Ans:D False negative

If anyone has a contradiction we can get the diagram on the thread and then argue...i
thought the answer was false negative..

133. Which among the following not used in diagnosis of insulinoma?


A. Fasting glucose test
B. Xylulose test
3. C peptide levels
4. Insulin / glucose ratio

Ans:B xylose test

which is for intestinal function..anyway here is sabiston reference for completion sake..

The pathognomonic finding is an inappropriately high (>5 mU/mL) level of serum insulin
during symptomatic hypoglycemia. A possible mechanism for this high level of insulin in
the face of hypoglycemia may be overexpression of the insulin splice variant. A
diagnostic ratio of blood insulin (in microunits per milliliter) to glucose (in milligrams per
deciliter) of greater than 0.4 or C peptide levels higher than 2 nmol/L have proved
valuable in diagnosis. The best way to induce hypoglycemia is with fasting: two thirds of
patients will experience hypoglycemic symptoms in 24 hours, and nearly all other
patients experience symptoms by 72 hours of fasting. Provocative tests, usually
involving tolbutamide or glucagon, have been used, but they may cause dangerously
profound hypoglycemia and are not generally necessary. Because cerebrocytes
metabolize only glucose, prolonged profound hypoglycemia may cause permanent brain
damage. Clinicians need to be alert to this problem when attempting to induce
hypoglycemia by fasting. Most important, preoperative fasting orders must be
accompanied by IV administration of glucose.
134. All are true about Nesidioblastosis except ?
A. Hypoglycemic episodes are seen
B. Occurs in adults more than children
C. Histopathology shows hyperplasia of islet cells
D. Diazoxide is used for treatment

Ans:B Occurs in adults more than children

The disorder later was called persistent hyperinsulinemic hypoglycemia of infancy (PHHI)

n PHHI, the histologic abnormalities in pancreatic structure are heterogeneous but can be
grouped into 2 broad categories: (1) focal adenomatous hyperplasia (found in one fourth to
one third of cases) and (2) a diffuse abnormality of the islets

Diazoxide (Hyperstat [IV], Proglycem [PO]) is an antihypertensive agent that relaxes smooth
muscle in the peripheral arterioles.
• Diazoxide is related to the thiazide class of drugs but has no diuretic action. It
promotes opening of the potassium adenosine triphosphate (ATP) channel, which
inhibits pancreatic secretion of insulin, stimulates glucose release from the liver, and
stimulates catecholamine release. (This effect is opposite that of the sulfonylurea
drugs used in diabetes mellitus, which close the ATP channel.)
• Diazoxide causes sodium and water retention and should be used cautiously in
patients with congestive heart failure or poor cardiac reserve. Hypertrichosis,
coarsening of the facies, decreased serum immunoglobulin G levels, and
hyperosmolar nonketotic comas have been reported with diazoxide, especially with
long-term use.
• Patients should be monitored for hypotension while using diazoxide, especially during
intravenous (IV) administration, because blood pressure may drop rapidly. Usually,
oral diazoxide is used for the treatment of hypoglycemia.
• Some authors recommend using chlorothiazide in conjunction with diazoxide for a
synergistic effect. Chlorothiazide activates a different potassium channel, and its
diuretic action helps counteract the salt and water retention associated with diazoxide
therapy.
135. Gold standard test for diagnosis of laryngopharyngeal reflux?
A. 24 hr double probe pH monitoring
B. Flexible endoscopy
C. Barium swallow
D. Laryngoscopy
Ans: A 24hr double probe pH monitoring

Reference:Sabiston 18th ed

The gold standard for diagnosing and quantifying acid reflux is the 24-hour pH test.[3]
The study is performed by placing a thin catheter containing one or more solid-state
electrodes in the esophagus. The electrodes are spaced 5 to 10 cm apart and are
capable of sensing fluctuations in the pH between 2 and 7. The electrodes are connected
to a data recorder that the patient wears for the period of observation. There is a digital
clock displayed on the recorder. When the patient has an event (e.g., heartburn, chest
pain, eructation), he or she is to record the event in a diary, noting the time on the
recorder ( Fig. 42-5 ).

136. Acoustic neuroma involves


A. Superior vestibular division of 8th cranial nerve
B. Auditory part of 8th cranial nerve
C. 7th cranial nerve
d. Inferior vestibular division of 8th cranial nerve

Ans: A Superior >Inferior---going with std texts

137. A patient had running nose and pain over medical aspect of eye. He later developed
chemosis,protosis and diplopia on abduction of right eye with congestion of optic disc.
What is the probable diagnosis?
A. Acute ethmoidal sinusitis
B. Orbital cellulitis
C. Cavernous sinus thrombosis
D. Orbital apex syndrome

Ans:C Cavernous sinus thrombosis(REPEAT)

138. An anesthesia resident was giving spinal anaesthesia when the patient had sudden
aphonia and loss of consciousness. What could have happened?
A. Total spinal
B. Partial spinal
C. Vaso vagal attack
D. Intra vessel injection

Ans:A Total spinal(REPEAT)

139. Urea cycle occurs in:


A. Liver
B. G.I.T.
C. Spleen
D. Kidney

Ans:A Liver

140. Right isomerism is?


A. Asplenia
B. Two spleens
C. One spleen
D. Polysplenia

Ans:A Asplenia

Right isomerism is also called asplenia syndrome

141. Definitive airway is all except?


A. Nasotracheal tube
B. Orotracheal tube
C. LMA(Laryngeal mask airway)
D. Cricothyroidectomy

Ans: c LMA

142. A primigravida at 37 weeks of gestation with loss of engagement and Cervix 1cm
dilated for the past 10hrs. What is management?
A. Sedate the patient and wait
B. LSCS
C. Amniotomy and augmentation with oxytocin
D. Induction with membrane rupture

Ans:A Sedate the patient and wait (REPEAT)

143. Regarding anterior choroidal artery syndrome, all are true except?
A. Hemipareisis
B. Hemisensory loss,
C. Involvement of anterior limb of internal capsule
D. Homonymous hemianopia

Ans:C Invovement of anterior limb of internal capsule

Reference:eMEDICINE and KEITH & MOORE CLINICAL ANATOMY

Posterior limb of internal capsule is supplied by the anterior choroidal artery

Anterior choroidal artery territory stroke features:


• Hemiparesis
• Hemianesthesia
• Homonymous hemianopia
144. A 15 day old baby came with history of seizures. Blood tests revealed Ca 5mg/dl,
PO4 9mg/dl, PTH 30pg/ml (n=10-60). What is the most probable diagnosis?
A. Pseudohypoparathyroidism
B. Vitamin D deficiency
C. Hyperparathyroidism
4. HIE

Ans:A Pseudohypoparathyroidism-

145. Thiamine deficiency causes decreased energy production because?


A. It is required for the process of transweramination
B. It is a co-factor in oxidative reduction
C. It is a co-enzyme for transwerketolase in pentose phosphate pathway
D. It is a co-enzyme for pyruvate dehydrogenase & alpha ketoglutarate
dehydrogenase
Ans:D It is a co-enzyme for pyruvate dehydrogenase & alpha ketoglutarate
dehydrogenase
Since kreb’s cycle is the main energy source for cell...it will be worst Hit in a thiamine
deficient state...

146. Mother to baby transmission of HIV can be minimised by all except?


A. Zidovudine
B. Vitamin A
C. Vaginal delivery
D. Avoidance of breast feeding

Ans: C Vaginal delivery(REPEAT)

147. True regarding leptospirosis is?


A. Rats are the min reservoirs
B. Fluroquinolones are the DOC
C. Person to person transrmission
D. Hepatorenal syndrome occurs in 50% cases

Ans: A Rats are the only reservoirs

Reference: Harrison 17th ed

Rodents, especially rats, are the most important reservoir, although other wild mammals
as well as domestic and farm animals may also harbor leptospires. These
microorganisms establish a symbiotic relationship with their host and can persist in the
renal tubules for years. Some serovars are generally associated with particular animals
(e.g., Icterohaemorrhagiae and Copenhageni with rats, Grippotyphosa with voles, Hardjo
with cattle, Canicola with dogs, and Pomona with pigs) but may occur in other animals
as well.

148. 18 year old male presents with hemetemesis, melena and splenomegaly. What is
the probable initial diagnosis?
A. NCPF
B. Cirrhosis
C. Malaria with DIC
D. Extra hepatic portal venous obstruction

Ans: D. Extra hepatic portal venous obstruction

149. Following are true about carbohydrate antigen except?


A. Memory
B. Poly clonal response
C. Poor immunogenicity
D. T cell independent immunity

Ans:A Memory(REPEAT)

150. Ideal age for surgery in unilateral undescended testis is?


A. 6 months
B. 12 months
C. 24 months
D. 36 months

Ans:B 12months OR A 6 months(convincing evidence for both answers


exist...........MOST DEBATED Q.....)

151. A 45 year old lady presented with DUB & USG finding of 8mm thick endometrium.
What is the next step?
A. Endometrial histopathology
B. Hysterectomy
C. OCP
D. Follow up

Ans”Endometrial histopathology(REPEAT)

152. Fallopian tube immotility is seen in?


A. Churg strauss syndrome
B. Kartagener's syndrome
C. Noonan syndrome
D. Turner syndrome

Ans:B Karteagener syndrome(REPEAT)

153. First structure to be fixed after amputation is?


A. Bone fixing
B. Arterial repair
C. Venous repair
D. Nerve repair

Ans:A Bone fixing

154. Poor prognostic factor for ALL is?


A. Hyperdiploidy
B. t(9;22) t(4;11)
C. 2-8 yrs of age
D. TLC < 50000

Ans:B . t(9;22) t(4;11)

155. Most potent activator of T cells?


A. B cells
B. Follicular dendritic cells
C. Mature dendritic cells
D. Macrophages

Ans:C Mature dendritic cells(REPEAT)

156.Not a disorder of protein misfolding ?


A. Alzheimer's disease
B. Tuberculosis
C. Cystic fibrosis
D. CJD

Ans: B TB

157. Aortic knuckle shadow on PA CXR, obliterated by consolidation of which portion of


lung?
upper lingula
lower lingula
apex of lower lobe
posterior part of upper lobe

Answer- Posterior part of upper lobe. (reference is still inadequate)

158. CT least accurate for:


a. 1 cm of aneurysm in hepatic artery
b.1 cm of lymph node in para-aortic region
c.1 cm of pancreas mass in tail
d. 1cm gall stone

Ans:D 1cm gall stones (reference is still inadequate)

159.Best investigation for bone metastases:


a.MRI
b.CT
c.bone scan
d. x ray

Ans:C Bone scan

160. Compliance is decreased in all except?


A. Pulmonary congestion
B. Emphysema
C. Decreased surfactant
D. Chronic bronchitis

Ans:D Chronic bronchitis

Compliance of the Lungs & Chest Wall


Compliance is also slightly greater when measured during deflation than when measured
during inflation. Consequently, it is more informative to examine the whole pressure–
volume curve. The curve is shifted downward and to the right (compliance is decreased)
by pulmonary congestion and interstitial pulmonary fibrosis (Figure 35–11). Pulmonary
fibrosis is a progressive restrictive airway disease of unknown cause in which
there is stiffening and scarring of the lung. The curve is shifted upward and to
the left (compliance is increased) in emphysema. It should be noted that
compliance is a static measure of lung and chest recoil. The resistance of the lung and
chest is the pressure difference required for a unit of air flow; this measurement, which
is dynamic rather than static, also takes into account the resistance to air flow in the
airways.
161. A 70yr old presents with intemittent jerks of recent origin, EEG showing bilateral
periodic spikes. What is the most probable diagnosis?
A. Hepes simplex encephalitis
B. Lewy body dementia
C. Alzheimer's
D. CJD

Ans:D CJD

Clinical Features
Nonspecific prodromal symptoms occur in about a third of patients with CJD
and may include fatigue, sleep disturbance, weight loss, headache, malaise, and ill-
defined pain. Most patients with CJD present with deficits in higher cortical function.
These deficits almost always progress over weeks or months to a state of profound
dementia characterized by memory loss, impaired judgment, and a decline in virtually all
aspects of intellectual function. A few patients present with either visual impairment or
cerebellar gait and coordination deficits. Frequently the cerebellar deficits are rapidly
followed by progressive dementia. Visual problems often begin with blurred vision and
diminished acuity, rapidly followed by dementia.
Other symptoms and signs include extrapyramidal dysfunction manifested as rigidity,
masklike facies, or choreoathetoid movements; pyramidal signs (usually mild); seizures
(usually major motor) and, less commonly, hypoesthesia; supranuclear gaze palsy; optic
atrophy; and vegetative signs such as changes in weight, temperature, sweating, or
menstruation.
Myoclonus
Most patients (~90%) with CJD exhibit myoclonus that appears at various
times throughout the illness. Unlike other involuntary movements, myoclonus persists
during sleep. Startle myoclonus elicited by loud sounds or bright lights is frequent. It is
important to stress that myoclonus is neither specific nor confined to CJD. Dementia with
myoclonus can also be due to Alzheimer's disease (AD) (Chap. 365), dementia with Lewy
bodies (Chap. 365), cryptococcal encephalitis (Chap. 195), or the myoclonic epilepsy
disorder Unverricht-Lundborg disease (Chap. 363).
Clinical Course
In documented cases of accidental transmission of CJD to humans, an incubation period
of 1.5–2.0 years preceded the development of clinical disease. In other cases, incubation
periods of up to 30 years have been suggested. Most patients with CJD live 6–12 months
after the onset of clinical signs and symptoms, whereas some live for up to 5 years.
Diagnosis
The constellation of dementia, myoclonus, and periodic electrical bursts in an
afebrile 60-year-old patient generally indicates CJD. Clinical abnormalities in CJD
are confined to the CNS. Fever, elevated sedimentation rate, leukocytosis in blood, or a
pleocytosis in cerebrospinal fluid (CSF) should alert the physician to another etiology to
explain the patient's CNS dysfunction.

157. Not a disorder of protein misfolding ?


A. Alzheimer's disease
B. Tuberculosis
C. Cystic fibrosis
D. CJD
Ans: B TB

The Cystic Fibrosis Gene: Mutational Spectra and Genotype-Phenotype


Correlation.
Since the CFTR gene was cloned in 1989, more than 1300 disease-associated mutations
have been identified. Various mutations can be grouped into six “classes” based on their
effect on the CFTR protein:
• Class I: Defective protein synthesis. These mutations are associated with
complete lack of CFTR protein at the apical surface of epithelial cells.
• Class II: Abnormal protein folding, processing, and trafficking. These
mutations result in defective processing of the protein from the endoplasmic
reticulum to the Golgi apparatus; the protein does not become fully folded and
glycosylated and is instead degraded before it reaches the cell surface. The most
common class II mutation is a deletion of three nucleotides coding for
phenylalanine at amino acid position 508 (ΔF508). Worldwide, this mutation can
be found in approximately 70% of cystic fibrosis patients. Class II mutations are
also associated with complete lack of CFTR protein at the apical surface of
epithelial cells.
• Class III: Defective regulation. Mutations in this class prevent activation of CFTR
by preventing ATP binding and hydrolysis, an essential prerequisite for ion
transport (see above). Thus, there is a normal amount of CFTR on the apical
surface, but it is nonfunctional.
• Class IV: Decreased conductance. These mutations typically occur in the
transmembrane domain of CFTR, which forms the ionic pore for chloride
transport. There is a normal amount of CFTR at the apical membrane, but with
reduced function. This class is usually associated with a milder phenotype.
• Class V: Reduced abundance. These mutations typically affect intronic splice sites
or the CFTR promoter, such that there is a reduced amount of normal protein. As
discussed subsequently, class V mutations are also associated with a milder
phenotype.
• Class VI: Altered regulation of separate ion channels. As previously described,
CFTR is involved in the regulation of multiple distinct cellular ion channels.
Mutations in this class affect the regulatory role of CFTR. In some cases, a given
mutation affects the conductance by CFTR as well as regulation of other ion
channels. For example, the ΔF508 mutation is both a class II and class VI
mutation.

162. Pulmonary toxicity is seen with?


A. Bleomycin
B. Cisplatin
C. Methotrexate
D. Actinomycin D

Ans:A Bleomycin

Reference: Goodman and Gilman pharmacology


The most serious adverse reaction to bleomycin is pulmonary toxicity, which
begins with a dry cough, fine rales, and diffuse basilar infiltrates on x-ray and may
progress to life-threatening pulmonary fibrosis. Radiologic changes may be
indistinguishable from interstitial infection or tumor, but may progress to dense fibrosis,
cavitation, atelectasis or lobar collapse, or even apparent consolidation. Approximately
5% to 10% of patients receiving bleomycin develop clinically apparent pulmonary
toxicity, and about 1% die of this complication.

163. A young male presented with history of fever and a nodule in the leg.
Histopathology of the nodule revealed foamy histiocytes and neutrophillic infiltrate in the
dermis. Most probable diagnosis is?
A. Sweet's syndrom
B. Rosai Dorfman disease
C. Erythema Nodosum Leprosum
D.erythema nodosum

Ans: D.erythema nodosum

164. Medical treatment for variceal bleed is by?


A. Octreotide
B. Pantaprazole
C. Desmopressin D.—

Ans: A Octreotide

Reference: Harrison Principles of Internal Medicine 17th ed

The medical management of acute variceal hemorrhage includes the use of


vasoconstricting agents, usually somatostatin or Octreotide. Vasopressin was
used in the past but is no longer commonly used. Balloon tamponade (Sengstaken-
Blakemore tube or Minnesota tube) can be used in patients who cannot get endoscopic
therapy immediately or who need stabilization prior to endoscopic therapy. Control of
bleeding can be achieved in the vast majority of cases; however, bleeding recurs in the
majority of patients if definitive endoscopic therapy has not been instituted. Octreotide,
a direct splanchnic vasoconstrictor, is given at dosages of 50–100 micg/h by continuous
infusion. Endoscopic intervention is employed as first-line treatment to control bleeding
acutely. Some endoscopists will use variceal injection therapy (sclerotherapy) as initial
therapy, particularly when bleeding is vigorous.

165. All are true statements regarding use of sodium fluoride in the treatment of
otosclerosis except?
A. It inhibits osteblastic activity
B. Used in active phase of otosclerosis when schwartz
C.Nephritis is a contraindication D.It decreases the release of osteolytic enzymes

Ans:A inhibits osteblastic activity(REPEAT)

166. A patient who was given primaquin develops hemolysis. Diagnosis is


A. Glucose 6 phosphate dehydrogenase deficiency
B. Glucose 6 phosphatase deficiency
C. --
D. –

Ans:A G6PD deficiency

167. 86 true regarding ranula?


A. It is also known as epulis
B. It is a cystic swelling in the floor of mouth
C. It is a type of thyroglossal cyst
D. It is a type of mucus retention cyst

Ans:B It is a cystic swelling in the floor of mouth

Reference: Robbins pathology 8th ed

A ranula is histologically identical to a mucocele. However, this term is reserved


for mucoceles that arise when the duct of the sublingual gland has been damaged. A
ranula can become extremely large and develop into a “plunging ranula” when it dissects
its way through the connective tissue stroma connecting the two bellies of the mylohyoid
muscle
168.Which of the following is not an evidence based treatment for menorrhagia?
A. Ethamsylate
B.OCP
C.Tamoxefene
D. –
Ans:A Ethamsylate(REPEAT)
169. -----one question has been numberd twice...the overall number shud tally........
170. 74 coarctation of aorta mc asso with
1.Bicuspid aortic valve
2.PDA
3. Aortic stenosis
4. --

Ans:A Bicuspid aortic valve(REPEAT)

171. Anaesthetic agent with vasoconstrictor is contraindicated in?


A. Finger block
B. Spinal block
C. Epidural block
D. Regional anaesthesia

Ans:A Finger block(REPEAT)

172. 63Auto-Rikshaw ran over a child’s thigh, there is a mark of the tyre tracks, it is an
A. Contact bruise
B. Patterned bruise
C. Imprint abrasion
D. Ectopic bruise
Ans:C Imprint abrasion(REPEAT)

173. Tolerance in opioids develops to all except?


A. Miosis
B. Analgesia
C. Euphoria
D. Nausea and vomiting

Ans: A Miosis

Reference:Katzung Pharmacology

MIOSIS
Constriction of the pupils is seen with virtually all opioid agonists. Miosis is a
pharmacologic action to which little or no tolerance develops ; thus, it is valuable
in the diagnosis of opioid overdose. Even in highly tolerant addicts, miosis is seen. This
action, which can be blocked by opioid antagonists, is mediated by parasympathetic
pathways, which, in turn, can be blocked by atropine.

174. The bifurcation of Common carotid artery is palpated at?


A. Upper border of cricoid cartilage
B. Upper border of thyroid cartilage
C. Hyoid bone
D. Cricothyroid membrane
Ans:B Upper border of thyroid cartilage

Reference:Snell’s Anatomy by regions

Common Carotid Artery


The right common carotid artery arises from the brachiocephalic artery behind the right
sternoclavicular joint (Figs. 11-57 and 11-59). The left artery arises from the arch of the
aorta in the superior mediastinum . The common carotid artery runs upward through the
neck under cover of the anterior border of the sternocleidomastoid muscle, from the
sternoclavicular joint to the upper border of the thyroid cartilage. Here it divides into
the external and internal carotid arteries.

175. Pregnant mother at 35 weeks of gestation. What drug can you not give her for
treatment of SLE?
A. Prednisolone
B. Methotrexate
C. Sulfsalazine
D. Hydroxychloroquine

Ans:B Methotrexate(REPEAT)

176. All are true about xanthogranulomatous inflammation except?


A. Presence of foamy macrophages
B. Associated with TB
C. Multinucleated giant cell
D. Presence of yellow Nodules
Ans:B Associated with TB(REPEAT)

177. Clue Cells are seen in :


A. Bacterial vaginosis
B. Vaginal candidiasis
C. Chlamydial vaginosis
D. Trichomoniasis

Ans: A Bacterial Vaginosis (REPEAT)

178. Which complement component is involved in both classical and alternate pathway?
A. C1
B. C2
C. C3
D. C4

Ans:C C3(REPEAT)

179. Which of the following are not associated with menstrual cycle?
A. Hormonal changes
B. Vaginal cytology changes
C. Estrus profile
D. Endometrial changes

Ans:C Estrous Profile (REPEAT)

180. Alkalanization of Urine is done during administration of which of the following


chemotherapeutic drugs?
A. ara-c
B. Mtx
C. cisplatin
D. Ifosfamide

Ans:A Methotrexate

Goldfrank's toxicologic emergencies


By Lewis R. Goldfrank, Neal Flomenbaum

Urinary alkalinisation with Sodium Bicarbonate is routinely used during high dose
Methotrexate cancer chemotherapy.

Methotrexate is predominantly excreted unchanged in urine.


It is poorly soluble in acidic urine.

The solubility and of Mtx can be increased and nephrotoxicity reduced by


Sodium bicarbonate administration.

181. The Progesterone in low dose OCP is? (contributed by dr.)


A.Norethisterone
B.Levonegestrol
C.Desgestrol
D.—

Ans: C Desogestrel > B Levonorgestrel(unless proved otherwise---was our best


ans from discussion)

Reference:Dutta Obstetrics

Reference:Dutta Obstetrics5th edition pg 465..

Low dose OCP:

Femilon(Infar)-desogestrol 0.15mg and Ethinyl estradiol 20mcg

182. Test of Pasterurized milk is performed by:


A. Phosphatase test
B. Coliform test
C. Catalase test
D. Methylene blue test

Ans:A Phosphatase test(REPEAT)

183. All are true about delirium tremens except?


A. Visual hallucinations
B. Coarse tremors
C. Third Nerve palsy
D. Altered consciousness

Ans:C Third Nerve Palsy

Reference:Kaplan and Sadock Synopsis of Psychiatry

The essential feature of the syndrome is delirium occurring within 1 week after a
person stops drinking or reduces the intake of alcohol. In addition to the symptoms of
delirium, the features of alcohol intoxication delirium include autonomic hyperactivity
such as tachycardia, diaphoresis, fever, anxiety, insomnia, and hypertension; perceptual
distortions, most frequently visual or tactile hallucinations; and fluctuating levels of
psychomotor activity, ranging from hyperexcitability to lethargy.

184. Main site of water absorption is:


A. Jejunum
B. Colon
C. Ileum
D. Stomach

Ans:A Jejunum (REPEAT)


185. Pentalogy of fallot has which one of following extra entities:
A. ASD
B. VSD
C. RVH
D. Pulmonary stenosis

Ans:A ASD(REPEAT)

186. All of the following are done in management of shoulder dystocia except?
A. Fundal pressure
B. Suprapubic pressure
C. McRoberts manoeuvre
D. Woods manoeuvre

Ans:A Fundal pressure(REPEAT)

187. About Human Development Index, all are true except?


A. Life expectancy at birth
B. Life expectancy at 1 year of age
C. Education
D. GDP

Ans: B. Life expectancy at 1 year of age(REPEAT)

188.A 50yr old patient presents with 2 yrs h/o recurrent abdominal pain, radiating to
back, relived only by parenteral analgesic. USG & CT confirmed the diagnosis
,appropriate procedure is?

A.vagotomy with Gastroduodenostomy


B.vagotomy with antrectomy
C.whipple procedure
D.Longitudinal pancreaticojejunostomy

Ans:D Longitudinal Pacreaticojejunostomy

The Diagnosis is Chronic Pancreatitis which was medically managed.

The treatment for it is Puestow’s longitudinal Pancreaticojejunostomy.

Reference:Sabiston Surgery 18th ed

Typically, patients with chronic pancreatitis have upper abdominal pain radiating
to the back. It can be constant or episodic and triggered by drinking alcohol or eating.
Repeated use of heating pads or hot water bottles to treat the chronic pain may result in
skin lesions (erythema ab igne) that define the distribution of the pain ( Fig. 55-5 ).
Some patients experience no pain.

For most patients with pain-ful chronic pancreatitis, intermittent or


persistent pain remains a major issue, and analgesics of increasing potency are
needed.

The two indications for surgical intervention are pain and concern about the
possible presence of cancer. After the diagnosis of chronic pancreatitis has been
established, surgical intervention is considered when (1) the pain is severe enough to
limit the patient's lifestyle or reduce productivity, and (2) the pain persists
despite complete abstinence from alcohol and administration of non-narcotic
analgesics.

Puestow and Gillesby, in 1958, described an operation that involved


longitudinally opening the entire duct and then invaginating the opened
pancreas into a Roux-en-Y loop of jejunum. This allowed for more complete
decompression but still required splenectomy. Later, Partington and Rochelle[37] modified
the Puestow procedure by creating a side-to-side anastomosis between the opened duct
and jejunum, thus eliminating the need for splenectomy ( Fig. 55-8 ). In appropriately
selected patients (i.e., those with large ducts and those with intraductal stones),
longitudinal pancreaticojejunostomy, performed according to the Partington and Rochelle
modification of the Puestow procedure, has been reported to result in immediate pain
relief in more than 80% of patients and long-term pain relief in roughly 60% of patients.
More recently, Ho and Frey[38] further modified the procedure by including removal of
part of the pancreatic head, thereby marsupializing the duct as it dives deeply in the
pancreas to reach the ampulla of Vater. This allows for an even more complete duct
decompression and a longer longitudinal pancreaticojejunostomy. Both short- and long-
term pain relief appear to be improved, and the procedure can be performed when the
duct is only moderately dilated.

189. A young lady presents with fever , dysuria and pain abdomen . Uncomplicated
acute cystitis was diagnosed . Which of these is false ?
A.Nitrate test positive
B.e.coli count was < 10 power 3
C.1 pus cell per 7 field
D.1 bacilli per field

Ans:?????????

190. In a 5 year old child the burn area corresponding to thesize of palm is equal to
A. 1% BSA
B. 5% BSA
C. 10% BSA
D.20% BSA

Ans:A 1%

Reference:Nelson Textbook Of Pediatrics 18th ed

ESTIMATION OF BODY SURFACE AREA FOR A BURN.


Appropriate burn charts for different childhood age groups should be used to accurately
estimate the extent of BSA burned. The volume of fluid needed in resuscitation is
calculated from the estimation of the extent and depth of burn surface. Mortality and
morbidity also depend on the extent and depth of the burn. The variable growth rate of
the head and extremities throughout childhood makes it necessary to use BSA charts,
such as that modified by Lund and Brower or the chart used at the Shriners Hospital in
Boston. The rule of nines used in adults may be used only in children older than age 14
yr or as a very rough estimate to institute therapy before transfer to a burn center. In
small burns of <10% of BSA, the rule of palm may be used, especially in outpatient
settings. The area from the wrist crease to the finger crease (the palm) in the
child equals 1% of the child's BSA.
191 What is the type of joint seen in the growth plate?
A. Fibrous
B. Primary cartilagenous
C. Secondary cartilagenous
D. Plane joint
Ans:B Primary cartilaginous joint(REPEAT)
192.The acid base status of a patient is as follows : pH - 7.45, pCO2 - 30 mm of Hg, pO2
- 105 mm of Hg. Patient has partially compensated?
A. Metabolic acidosis
B. Metabolic alkalosis
C. Respiratory acidosis
D. Respiratory alkalosis
Ans:D Respiratory alkalosis(No explanation needed)
193 A female presents with sings of meningitis. CSF shows gram positive bacilli. It is
most probably?
A. Listeria
B. Haemophilus influenzae
C. Pneumococcus
D. Staphylococcus
Ans:A Listeria
194.A 50 yr lady has history of sprained ankle 2 months back followed by recovery. She
now complains of severe pain in that ankle with inability to flex that foot. Physician notes
edema and shiny skin in local examination. What is the probable diagnosis:
A. Fibromyalgia
B. Complex regional pain syndrome 1
C. Complex regional pain syndrome 2
D. Peripheral neuropathy
Ans:B Complex regional pain syndrome 1
Reference:Harrison Principles Of Internal Medicine 17th ed
The failure to identify a primary role of the ANS in the pathogenesis of these
disorders has resulted in a change of nomenclature. Complex regional pain syndrome
(CRPS) types I and II are now used in place of reflex sympathetic dystrophy (RSD) and
causalgia, respectively.
CRPS type I is a regional pain syndrome that usually develops after tissue
trauma. Examples of associated trauma include myocardial infarction, minor shoulder or
limb injury, and stroke. Allodynia (the perception of a nonpainful stimulus as painful),
hyperpathia (an exaggerated pain response to a painful stimulus), and spontaneous pain
occur. The symptoms are unrelated to the severity of the initial trauma and are not
confined to the distribution of a single peripheral nerve. CRPS type II is a regional pain
syndrome that develops after injury to a peripheral nerve, usually a major nerve trunk.
Spontaneous pain initially develops within the territory of the affected nerve but
eventually may spread outside the nerve distribution.
Pain is the primary clinical feature of CRPS. Vasomotor dysfunction, sudomotor
abnormalities, or focal edema may occur alone or in combination but must be present for
diagnosis. Limb pain syndromes that do not meet these criteria are best classified as
"limb pain—not otherwise specified." In CRPS, localized sweating (increased resting
sweat output) and changes in blood flow may produce temperature differences between
affected and unaffected limbs.
CRPS type I (RSD) has classically been divided into three clinical phases but is
now considered to be more variable. Phase I consists of pain and swelling in the distal
extremity occurring within weeks to 3 months after the precipitating event. The pain is
diffuse, spontaneous, and either burning, throbbing, or aching in quality. The involved
extremity is warm and edematous, and the joints are tender. Increased sweating and
hair growth develop. In phase II (3–6 months after onset), thin, shiny, cool skin
appears. After an additional 3–6 months (phase III), atrophy of the skin and
subcutaneous tissue plus flexion contractures complete the clinical picture.
The natural history of typical CRPS may be more benign than reflected in the
literature. A variety of surgical and medical treatments have been developed, with
conflicting reports of efficacy. Clinical trials suggest that early mobilization with physical
therapy or a brief course of glucocorticoids may be helpful for CRPS type I. Other
medical treatments include the use of adrenergic blockers, nonsteroidal anti-
inflammatory drugs, calcium channel blockers, phenytoin, opioids, and calcitonin.
Stellate ganglion blockade is a commonly used invasive therapeutic technique that often
provides temporary pain relief, but the efficacy of repetitive blocks is uncertain.

194.Drug given for bone megakaryocyte stimulation in patient of thrombocytopenia..?


1.filgastrim
2.oprevelkin
3.--
4.—
Ans: B Oprelvekin
Reference:Goodman and Gilman Pharmacology
Recombinant human interleukin-11 oprelvekin (NEUMEGA) is a bacterially
derived, 19,000-dalton polypeptide of 177 amino acids that differs from the native
protein only because it lacks the amino terminal proline residue and is not glycosylated.
The recombinant protein has a 7-hour half-life after subcutaneous injection. In normal
subjects, daily administration of oprelvekin leads to a thrombopoietic response in 5 to 9
days.
The drug is available in single-use vials containing 5 mg and is administered to
patients at 25 to 50 micg/kg per day subcutaneously. Oprelvekin is approved for use in
patients undergoing chemotherapy for nonmyeloid malignancies that displayed severe
thrombocytopenia (platelet count <20,000/microL) on a prior cycle of the same
chemotherapy, and is administered until the platelet count returns to more than
100,000/micL. The major complications of therapy are fluid retention and other
associated cardiac symptoms, such as tachycardia, palpitation, edema, and shortness of
breath; this is a significant concern in elderly patients and often requires concomitant
therapy with diuretics. Fluid retention reverses upon drug discontinuation, but volume
status should be carefully monitored in elderly patients, those with a history of heart
failure, or those with preexisting fluid collections in the pleura, pericardium, or peritoneal
cavity. Also reported are blurred vision, injection-site rash or erythema, and
paresthesias.

196. A man presents with maculopapular rash 2weeks after having a painless
genital.Causative organism of the condition is:
A.treponema pallidum
B.chlamydia
C.c.granulomatis
D.H.ducreyi
Ans:A Treponema pallidum(Its syphilis—anyway shall upload references later)
197. A patient with stab injury presents with with omentum protruding in the umbilical
area ,vitals stable.The next step in the management of the patient is: (CONTRIBUTED BY
dr.lucifer)
A.FAST
B.LAPAROTOMY
C.WOUND EXPLORATION
D.CECT ABDOMEN
Ans:B Laparotomy>/C Wound Exploration

Reference:Sabiston Surgery 18th ed eMEDICINE and Fundamentals of Surgical Practice


By Andrew N. Kingsnorth, Aljafri A. Majid

Abdominal Trauma
The abdomen is frequently injured after both blunt and penetrating trauma.
Approximately 25% of all trauma victims will require abdominal exploration. Clinical
evaluation of the abdomen by means of physical examination is inadequate to identify
intra-abdominal injuries because of the high number of patients with altered mental
status secondary to head trauma, alcohol, or drugs and because of the inaccessibility of
the pelvic, upper abdominal, and retroperitoneal organs to palpation. For these reasons,
several diagnostic modalities have evolved during the past 3 decades, including
diagnostic peritoneal lavage (DPL), ultrasound, CT, and laparoscopy, all of which have
advantages, disadvantages, and limitations.
The development of more modern technology, experience, and invasiveness have been
the most important determinants of the use of diagnostic methods for abdominal
trauma. In modern trauma centers in the 21st century, better noninvasive technology
favors the use of ultrasound and CT in the evaluation of trauma victims.
Mechanism of Injury
Blunt trauma secondary to motor vehicle accidents, motorcycle accidents, falls, assaults,
and striking of pedestrians remains the most frequent mechanism of abdominal injury.
Penetrating abdominal wounds are usually caused by either gunshot or stab wounds and
by a significantly smaller number of shotgun wounds.
Based on the high frequency of intra-abdominal organ injury after gunshot wounds,
mandatory abdominal exploration, with the rare exception of tangential and superficial
wound trajectories restricted to the right upper quadrant, remains the standard form of
management. Stab wounds to the abdomen, however, carry a significantly lower risk of
intra-abdominal organ injury than do gunshot wounds, and several studies have recently
favored a more selective approach, as opposed to mandatory exploratory laparotomy.
The impetus for nonoperative management of solid organ injury in stable blunt trauma
patients has expanded to penetrating trauma as well. With improved imaging, more
stable patients sustaining a single solid organ injury after stab and gunshot wounds to
the abdomen will be treated conservatively.
In children, besides the aforementioned mechanisms of injury, child abuse and trauma
secondary to recreational activities such as bicycling, swimming, and roller skating
should also be considered.
Diagnosis
The history of the traumatic event is particularly important in determining the likelihood
of an intra-abdominal organ injury. All possible information should be obtained from the
prehospital personnel, including the mechanism of injury, the height of a fall, damage to
the interior and exterior of a vehicle in a motor vehicle accident, other deaths at the
scene, ejection, vital signs, mental status, the presence of external bleeding, the type of
weapon, and other pertinent data.
On arrival at the hospital, the history and physical examination are usually accurate in
determining intra-abdominal injury in an awake and responsive patient, although the
limitations of physical examination are significant. Many patients with moderate intra-
abdominal bleeding will be in a compensated hemodynamic condition and will not have
peritoneal signs. Furthermore, retroperitoneal and pelvic injuries cannot be ruled out on
the basis of only physical findings. We believe that an objective abdominal evaluation is
necessary and should be performed by any of the available diagnostic modalities, in
addition to the physical examination. The test of choice depends on the hemodynamic
stability of the patient and the severity of associated injuries.
Hemodynamically stable patients sustaining blunt trauma are adequately evaluated by
abdominal ultrasound or CT, unless other severe injuries take priority and the patient
needs to go to the operating room before the objective abdominal evaluation. In such
instances, DPL or focused abdominal sonography for trauma (FAST) is usually performed
in the operating room to rule out intra-abdominal bleeding requiring immediate surgical
exploration.
Hemodynamically stable blunt trauma patients are evaluated by ultrasound in the
resuscitation room, if available, or by DPL to rule out intra-abdominal injuries as the
source of blood loss and hypotension.
Hypotensive patients with isolated penetrating abdominal trauma who are hypotensive
or in shock or have peritoneal signs should go to the operating room despite the
mechanism of injury. Stab wound victims without peritoneal signs, evisceration, or
hypotension benefit from wound exploration and DPL. Gunshot wound victims should
generally undergo exploration.

Fundamentals of Surgical Practice By Andrew N. Kingsnorth, Aljafri A. Majid


Indications for Operation in Anterior abdominal stab wounds:
Shock ; Peritonitis and Visceral Protrusion(OMENTUM EXCLUDED)
In Equivocal cases the investigation of choice is Exploration of wound to find if the
peritoneum is breached.

eMEDICINE reference:
Abdominal stab wound exploration forms part of a strategy developed by surgeons to allow a more
selective approach. In asymptomatic patients with stab wounds to the anterior abdomen, 2 methods
are widely used to help determine the need for laparotomy:
• Abdominal stab wound exploration (Subsequent diagnostic peritoneal lavage [DPL], serial
clinical evaluation, or both are used to further assess patients in whom an exploration cannot
definitively exclude peritoneal penetration.)
• Serial clinical evaluation

198. Site not affected in posterior cerebral artery infarct is?


A. Midbrain
B. Pons
C. Thalamus
D. Striate Cortex
Ans:B Pons(Not referenced)

199. A poison Illuminous, translucent, waxy


A. Yellow phosphorus
B. Arsenic
C. Thalium
D. ?
Ans:A Yellow phosphorous
200 . A pt comes with history of unresponsive fever n cough. xray shows
pneumonia,sputum reveals aerobic, gram positive and partially acid fast branching
filaments.Causative organism is?

A. Actinomycosis
B. Nocardiosis
C. Aspergillus
D.—
Ans:B Nocardiosis
Reference: Harrison Principles Of Internal Medicine 17th ed:
Pneumonia, the most common form of nocardial disease in the respiratory tract,
is typically subacute; symptoms have usually been present for days or weeks at
presentation. The onset is occasionally more acute in immunosuppressed patients.
Cough is prominent and produces small amounts of thick, purulent sputum that is not
malodorous. Fever, anorexia, weight loss, and malaise are common; dyspnea, pleuritic
pain, and hemoptysis are less common. Remissions and exacerbations over several
weeks are frequent. Roentgenographic patterns vary, but some are highly suggestive of
nocardial pneumonia. Infiltrates vary in size and are typically of at least moderate
density. Single or multiple nodules are common , sometimes suggesting tumor
metastases. Infiltrates and nodules tend to cavitate (Fig. 155-2). Empyema is present in
one-third of cases.
The first step in diagnosis is examination of sputum or pus for crooked, branching,
beaded, gram-positive filaments 1 microm wide and up to 50microm long . Most
nocardiae are acid-fast in direct smears if a weak acid is used for decolorization (e.g., in
the modified Kinyoun, Ziehl-Neelsen, and Fite-Faraco methods). The organisms often
take up silver stains.
Nocardiae grow relatively slowly; colonies may take up to 2 weeks to appear and may
not develop their characteristic appearance for up to 4 weeks. Several blood culture
systems support nocardial growth. Yield in manual systems is enhanced when blood
cultures are incubated aerobically for up to 4 weeks and when blind subcultures are
performed. Nocardial growth is so different from that of more common pathogens that
the laboratory should be alerted when nocardiosis is suspected in order to maximize the
likelihood of isolation. Since nocardiae are among the few aerobic microorganisms that
use paraffin as a carbon source, paraffin baiting can be used to isolate the organisms
from mixed cultures.
201. Which among the following is the best inotrope drug for use in right heart failure
with primary pulmonary hypertension?
A. Dobutamine
B. digoxin
C. Halothane
D. Milrinone
Ans:D Milrinone
Ans:Goodman and Gilman Pharmacology
Inamrinone and Milrinone
Parenteral formulations of inamrinone (previous name amrinone) and milrinone have
been approved for short-term support of the circulation in advanced heart failure. Both
drugs are bipyridine derivatives and relatively selective inhibitors of PDE3, the cyclic
GMP–inhibited cyclic AMP PDE.
These drugs cause direct stimulation of myocardial contractility and acceleration of
myocardial relaxation. In addition, they cause balanced arterial and venous dilation with
a consequent fall in systemic and pulmonary vascular resistances, and left and right
heart filling pressures.
Cardiac output increases due to the stimulation of myocardial contractility and the
decrease in left ventricular afterload.
As a result of this dual mechanism of action, the increase in cardiac output with
milrinone is greater than that seen with nitroprusside at doses that produce comparable
reductions of systemic resistance. Conversely, the arterial and venous dilator effects of
milrinone are greater than those of dobutamine at doses that produce comparable
increases in cardiac output .
202. Which of the following helps in generating oxygen burst for killing bacteria within
neurophils ?
A. Superoxide dismutase
B. Oxidase
C. Peroxidase
D. Glutathione reductase
Ans:B Oxidase(REPEAT)
203. Main blood supply of neck of femur?
A. Lateral circumflex femoral
B. Medial circumflex femoral
C. Profunda femoris answer
D. Popliteal artery
Ans: B Medial circumflex femoral artery
Reference:Snell’s Clinical anatomy by regions
Blood Supply to the Femoral Head and Neck Fractures

Anatomic knowledge of the blood supply to the femoral head explains why avascular
necrosis of the head can occur after fractures of the neck of the femur. In the young, the
epiphysis of the head is supplied by a small branch of the obturator artery, which passes
to the head along the ligament of the femoral head.

The upper part of the neck of the femur receives a profuse blood supply from the medial
femoral circumflex artery. These branches pierce the capsule and ascend the neck deep
to the synovial membrane.

As long as the epiphyseal cartilage remains, no communication occurs between the two
sources of blood. In the adult, after the epiphyseal cartilage disappears, an anastomosis
between the two sources of blood supply is established. Fractures of the femoral neck
interfere with or completely interrupt the blood supply from the root of the femoral neck
to the femoral head. The scant blood flow along the small artery that accompanies the
round ligament may be insufficient to sustain the viability of the femoral head, and
ischemic necrosis gradually takes place.
204. All are actions of muscarinic antagonist except?
A. Decreases gastric secretion
B. Prolongs a-v conduction
C. Decreases respiratory secretions
D. Contraction of radial muscles of iris
Ans:D Contraction of radial muscles of Iris
On seeing the Q first it appears as if all 4 options are correct....Anyway cholinergic
system contracts circular muscle of iris...an Antimuscarinic agen will block this and this
inturn leads to pupilary dilatation....contraction of radial muscles of Iris is a function of
sympathetic system.
205. All are used in the treatment of hot flushes except?
A. Tamoxifene
B. Venlafaxine
C.
D.
Ans:A Tamoxifen(REPEAT)
206. OPTIONS UNCLEAR SO 2 VERSIONS HAVE BEEN PUT UP:
VERSION 1: Pearson's skewness coefficient is?
A. (Mean-mode)/SD
B. Mode-mean/SD
C. SD/mean-mode
D. SD/mode-mean

Ans- mean-mode/SD
VERSION 2: Pearson's skewness coefficient is?
A. (Mean-median)/SD
B. Median-mean/SD
C. SD/mean-median
D. SD/median-mean
Ans:A (Mean-median)/SD
Reference:WIKIPEDIA
Pearson's skewness coefficients

Karl Pearson suggested simpler calculations as a measure of skewness:[5] the Pearson mode or first
skewness coefficient[6], defined by

 (mean − mode) / standard deviation,

as well as Pearson's median or second skewness coefficient,[7], defined by

 3 (mean − median) / standard deviation.

207. Best treatment option for genuine stress incontinence?


A. Burch colposuspension
B. Kelly’s procedure
C. Sling operation
D. Tension free vaginal taping
Ans:A Burch colposuspension
Reference:Berek and Novak Gynaecology
In 1997, the American Urological Association convened a clinical guidelines panel
to analyze published outcomes data on surgical procedures to treat female stress urinary
incontinence and to produce practice recommendations to guide surgical decision–
making . The panel concluded that colposuspensions (e.g., Burch, Marshall–
Marchetti–Krantz [MMK]) and slings were more effective than transvaginal
needle suspensions or anterior repairs for long–term success (48 month
cure/dry rates). The median probability estimates for cure/dry rates at 48
months and longer were 84% (95% CI, 79%–88%) for colposuspensions and
83% (95% CI, 75%–88%) for sling procedures, compared with 67% (95% CI,
53%–79%) for transvaginal needle suspensions and 61% (95% CI, 47%–
72%) for anterior repairs.
208. Lines of blaschko are:
A. Lymphatics
B. Blood vessel
C. Nerves
D. Lines of development
Ans:D Lines of development

Reference: Avery's neonatology: pathophysiology & management of the


newborn By Gordon B. Avery, Mhairi G. MacDonald, Mary M. K. Seshia, Martha
D. Mullett
“During development, skin and its structures are thought to migrate along the
"lines of Blaschko" . Most authorities believe thatBlaschko lines are an expression of
epidermal and not dermal migration”

209. All are true about meglitinides except?


A. Decreases post parandial hyperglycemia
B. Hypoglycemia less common than sulfonylureas
C. It decreases insulin resistance
D. It acts by releasing insulin .
Ans:C It decreases Insulin resistance
INSULIN SECRETAGOGUES: MEGLITINIDES
The meglitinides are a relatively new class of insulin secretagogues. Repaglinide, the
first member of the group, was approved for clinical use in 1998 (Table 41–7). These
drugs modulate B-cell insulin release by regulating potassium efflux through the
potassium channels previously discussed. There is overlap with the sulfonylureas in their
molecular sites of action because the meglitinides have two binding sites in common
with the sulfonylureas and one unique binding site.

Repaglinide has a very fast onset of action, with a peak concentration and peak effect
within approximately 1 hour after ingestion, but the duration of action is 5–8 hours. It is
hepatically cleared by CYP3A4 with a plasma half-life of 1 hour. Because of its rapid
onset, repaglinide is indicated for use in controlling postprandial glucose excursions. The
drug should be taken just before each meal in doses of 0.25–4 mg (maximum, 16
mg/d); hypoglycemia is a risk if the meal is delayed or skipped or contains inadequate
carbohydrate. This drug should be used cautiously in individuals with renal and hepatic
impairment. Repaglinide is approved as monotherapy or in combination with biguanides.
There is no sulfur in its structure, so repaglinide may be used in type 2 diabetic
individuals with sulfur or sulfonylurea allergy.

INSULIN SECRETAGOGUE: D-PHENYLALANINE DERIVATIVE


Nateglinide, a D-phenylalanine derivative, is the latest insulin secretagogue to become
clinically available. Nateglinide stimulates very rapid and transient release of insulin from
B cells through closure of the ATP-sensitive K+ channel. It also partially restores initial
insulin release in response to an intravenous glucose tolerance test. This may be a
significant advantage of the drug because type 2 diabetes is associated with loss of this
initial insulin response. The restoration of more normal insulin secretion may suppress
glucagon release early in the meal and result in less endogenous or hepatic glucose
production. Nateglinide may have a special role in the treatment of individuals with
isolated postprandial hyperglycemia, but it has minimal effect on overnight or fasting
glucose levels. Nateglinide is efficacious when given alone or in combination with
nonsecretagogue oral agents (such as metformin). In contrast to other insulin
secretagogues, dose titration is not required.

Nateglinide is ingested just before meals. It is absorbed within 20 minutes after oral
administration with a time to peak concentration of less than 1 hour and is hepatically
metabolized by CYP2C9 and CYP3A4 with a half-life of 1.5 hours. The overall duration of
action is less than 4 hours.

Nateglinide amplifies the insulin secretory response to a glucose load but has a markedly
diminished effect in the presence of normoglycemia. The incidence of hypoglycemia may
be the lowest of all the secretagogues, and it has the advantage of being safe in
individuals with very reduced renal function.

210. All are true regarding serotonin syndrome except?


A. It is not idiosyncratic and unpredictable
B. Dantrolene is the drug of choice for the condition
C. --
D. –
Ans:B Dantrolene is the drug of choice for the condition(A Query on the exact
wordings of the 2nd option exists—harrison does mention dandrolene in
treatment of serotonin syndrome,but its not the drug of choice)
Reference:WIKIPEDIA
Management is based primarily on stopping the usage of the precipitating drugs, the administration of
serotonin antagonists such as cyproheptadine, and supportive care including the control of agitation,
the control of autonomic instability, and the control of hyperthermia.
Additionally, those who ingest large doses serotonergic agents may benefit from gastrointestinal
decontamination with activated charcoal if it can be administered within an hour of overdose.
The intensity of therapy depends on the severity of symptoms. If the symptoms are mild, treatment
may only consist of discontinuation of the offending medication or medications, offering supportive
measures, giving benzodiazepines for myoclonus, and waiting for the symptoms to resolve.
Moderate cases should have all thermal and cardiorespiratory abnormalities corrected and can
benefit from serotonin antagonists. The serotonin antagonist cyproheptadine is the recommended
initial therapy, although there have been no controlled trials demonstrating its efficacy for serotonin
syndrome.[3][46][47] Despite the absence of controlled trials, there are a number of case reports detailing
apparent improvement after patients have been administered cyproheptadine.[3] Animal experiments
also suggest a benefit from serotonin antagonists.
Cyproheptadine is only available as tablets and therefore can only be administered orally or via
a nasogastric tube; it is unlikely to be effective in patients administered activated charcoal and has
limited use in severe cases.[3] Additional pharmacological treatment for severe case includes
administering atypical antipsychotic drugs with serotonin antagonist activity such as olanzapine.
[1]
Critically ill patients should receive the above therapies as well as sedation or neuromuscular
paralysis.[1] Patient who have autonomic instability such as low blood pressure require treatment with
direct-acting sympathomimetics such as epinephrine, norepinephrine, orphenylephrine.[1] Conversely,
hypertension or tachycardia can be treated with short-acting antihypertensive drugs such
as nitroprusside or esmolol; longer acting drugs such as propranolol should be avoided as they may
lead to hypotension and shock.
211 All of the following are neuronal tumours except? (QUOTED by imbackagain)
A. Gangliocytoma
B. Ganglioglioma
C. Neuroblastoma
D. Ependymoma
Ans:D Ependymoma

REF: Robbins's.. Page 1348


I got hold of old edition thus giving the page number of old edition but yo ucan find it
under heading of NEURONAL TUMOURS in robbins.. at present edition that I got hold of
is old edition page 1348.. But as I said under heading of NEuronal tumours it says this..

" Several types of Cns tumors contain mature appearing neurons(ganglion cell these
might be complete population(GANGLIOCYTOMa) or sometimes it might be mixed with
glial tissue ( GANGLIOGLIOMA).. OTHERS INCLUDE CEREBRAL NEUROBLASTOMA
,CENTRAL NEUROCYTOMA AND DYSEMBRYONIC NEUROEPITHELIAL TUMOR
212. Orthopnoea in right heart failure develops due to?
A. Reservoir function of pulmonary veins
B. Reservoir function of leg veins
C. --
D. –-
Ans:B Reservoir function of leg veins
Reference:BRAUNWALD’s PRINCIPLES OF CARDIOLOGY
ORTHOPNEA:
Orthopnea is defined as dyspnea that occurs in the recumbent position and is usually a
later manifestation of heart failure than exertional dyspnea.
Orthopnea is generally relieved by sitting upright or by sleeping with additional pillows.
It results from the redistribution of fluid from the splanchnic circulation and lower
extremities into the central circulation during recumbency, with a resultant increase in
pulmonary capillary pressures.
Nocturnal cough is a frequent manifestation of this process, and is a frequently
overlooked symptom of heart failure. Although orthopnea is a relatively specific
symptom of heart failure, it may occur in pulmonary patients with abdominal obesity or
ascites, and in pulmonary patients whose lung mechanics favor an upright posture.
213. Commonest cause for bilateral proptosis in children?
A. Cavernous haemangioma
B. Chloroma
C. Histiocytoma
D. ---
Ans:D Chloroma
Reference: “Proptosis in children: An Approach”
http://medind.nic.in/ias/t04/s2/iast04s2p33g.pdf

The most common causes of Bilateral ptosis in children is granulocytic sarcoma in


AML/MDS and metastatic neuroblastoma
Most common cause of malignant proptosis in children:
In india—Retinoblastoma
Western countries-Rhabdomyosarcoma
214.Most common malignant cause of proptosis in children?
A. A. Cavernous haemangioma
B. Rhabdomyosarcoma
C. ALL
D. AML
Ans:B Rhabdomyosarcoma
215. Blood examination of a patient revealed Ca 12.5 and Alkaline Po4 900IU/L. Which
of the following investigations need not be done?
A. Urine microscopy
B. PTH reassessment
C. Vitamin D levels
D. –
Ans:Urine microscopy
216. Most useful indicator for acute illness
A.case fatalityrate
B. --
C. --
D. –-
Ans:A Case fatality rate(REPEAT)
217.Endolymphatic Hydrops is seen in
A. Meniere’s disease
B.Otosclerosis
C.—
D.—
Ans:A Meniere’s disease(No reference needed)
218. about contrast radiography true is
a.3 parts of iodine with 2 parts of solvent
b.injection into artery is associated with 1/3 times more complication than injection into
a vein
c.Test dose should be performed if a contrast reaction is suspected
d.—
Ans a.3 parts of iodine with 2 parts of solvent

219. A female with XO genotype and Primary amenorrhoea most likely diagnosis is?
A. Gonadal dysgenesis
B. Androgen insensitivity syndrome.
C. MRKH
D. –
Ans:A Gonadal dysgenesis
220. 1.Cicatrising alopecia with perifolicular blue-gray pathches (??) is most commonly
associated with
a. Nail dystrophy
b. Whitish lesion in the buccal mucosa
c. Arthritis

d. Discoid Plaques in the face


Ans:??????????
Reference:
http://www.scielo.br/pdf/abd/v85n2/en_08.pdf
http://www.jewds.eg.net/pdf/vol_7_1/1.pdf
221.Transfer of an amino group from an amino acid to an alpha keto acid is done by?
A. Transaminases
B. Aminases
C. Transketoses
D. –
Ans:A Transaminases(no references needed)
222. True about gastric carcinoma is?
A. Occult bleeding in stool is not seen
B. Often associated with achlorhydria/hypochlorhydria
C. Always squamous cell carcinoma
D. Radiosensitive
Ans:B Often associated with achlorhydria/hypochlorhydria
Reference:Schwartz surgery
Most patients who are diagnosed with gastric cancer in the United States have
advanced stage III or IV disease at the time of diagnosis. The most common symptoms
are weight loss and decreased food intake due to anorexia and early satiety. Abdominal
pain (usually not severe and often ignored) also is common. Other symptoms include
nausea, vomiting, and bloating. Acute GI bleeding is somewhat unusual (5%), but
chronic occult blood loss is common and manifests as iron deficiency anemia and heme-
positive stool. Dysphagia is common if the tumor involves the cardia of the stomach.
Paraneoplastic syndromes such as Trousseau's syndrome (thrombophlebitis), acanthosis
nigricans (hyperpigmentation of the axilla and groin), or peripheral neuropathy are
rarely present.
Chronic atrophic gastritis is by far the most common precursor for gastric cancer,
particularly the intestinal subtype . The prevalence of atrophic gastritis is higher in older
age groups, but it is also common in younger people in areas with a high incidence of
gastric cancer. In many patients, it is likely that H. pylori is involved in the pathogenesis
of atrophic gastritis. Correa described three distinct patterns of chronic atrophic gastritis:
autoimmune (involves the acid-secreting proximal stomach), hypersecretory (involving
the distal stomach), and environmental (involving multiple random areas at the junction
of the oxyntic and antral mucosa).
223. A boy gets hit by a tennis ball in the eye following which he has complaints of
decreased vision. Which of the following tells that blunt injury is due to the ball.
1. Optic neuritis
B. Pars planitis
C. Vitreous base detachment
D. Equatorial edema
Ans: C Vitreous base detachment

Reference: Vitreoretinal disease: the essentials By Carl D. Regillo, Gary C.


Brown, Harry W. Flynn
“The presence of Vitreous base avulsion is thought to be pathognomonic of blunt ocular
trauma”

224. False about C.diphtheriae is?


A. Toxin production is chromosome mediated
B. org cnfd by toxin production
C. Toxic to heart and neurons
D. toxin blocks protein synthesis
Ans:A Toxin production is chromosome mediated

225. True regarding drug resistance of MRSA? (was a long stem though)
A. Penicillinase enzyme production
B. Due to change in penicillin binding receptors
C. plasmid mediated
D. Treated with amoxicillin clavulanic acid

Ans:B Due to change in penicillin binding receptors

226. Muscular component of dorsal aorta develops from?


A. Axial mesoderm
B. Paraxial mesoderm
C. Intermediate mesoderm
D. Lateral plate mesoderm

Ans: B paraxial mesoderm

Reference: DEVELOPMENTAL ORIGIN AND MOLECULAR REGULATION OF VASCULAR


SMOOTH MUSCLE CELLS

Per Wasteson Institute of Biomedicine, Department of Medical Biochemistry and Cell


Biology, Wallenberg Laboratory for Cardiovascular Research, Sahlgrenska Academy,
University of Gothenburg, Göteborg, Sweden, 2008

227. Dental numbering is done by all except?


A. FDI two digit system
B. Anatomic and diagramatic charting
C. Palmer notation
D. Acrogram

Ans: option D acrogram

228. False about pneumococcus is?


A. Capsule aids in virulence
B. Commonest cause of otitis media and pneumonia
C. Meningitis caused by it is milder than other
D. ---

Ans:C Meningitis caused by it is milder than other organisms

229. False regarding Japanese encephalitis is:


A. Epidemic is 2-3 cases in a village
B. Mosquito bite is always associated with disease
C. 70 in infants
D. Apparent and nonapparent ratio 1:100

Ans:B Mosquito bite is always associated with disease

230. True in keto acidosis is..


A. Decreased HCO3
B. Increased levels of lactate
C. --
D. –

Ans:A Decreased Hco3

231. A 6 week old male infant was brought in a state of dehydration and shock.
Examination revealed hyper pigmentation over the body with normal external genitalia.
Blood tests revealed hypoglycemia, Na - 124 mEq/L and K - 7 mEq/L. What is the
probable diagnosis ?
A. Congenital adrenal hyperplasia
B. Adrenal haemorrhage and shock
C. Acute gastroenteritis with dehydration
D. –

Ans:A CAH(modified REPEAT)

232. All of the following are true regarding diabetes mellitus except?
A. Insulin is not used in type 2 diabetes
B. Sliding scale regimen is used in hospitals
C. --
D. –

Ans:A Insulin is not used in type 2 DM

233. Which among the following is an early sign of magnesium toxicity?


A. Loss of deep tendon reflexes
B. Respiratory depression
C. Cardiac arrest
D. Decreased urine output

Ans:A Loss of DTR

234. All of the following are affected in low radial nerve palsy except?
A. Extensor carpi radialis longus
B. Extensor carpi radialis brevis
C. Finger extensors
D. Sensation on dorsum of hand

Ans:A ECRL(REPEAT)

235. A man connected to a body plethysmograph exhales against a closed glottis. What
will be the finding?
A. The pressure in both the lungs and the box increases
B. The pressure in both the lungs and the box decreases
C. The pressure in the lungs decreases, but that in the box increases
D. The pressure in the lungs increases, but that in the box decrease

Ans: D. The pressure in the lungs increases, but that in the box decrease

236. A patient presents with fever and abdominal pain. Clinical examination reveals
hepatomegaly extending 4 finger breadths below the costal margin. USG reveals a
5cm*5cm*4cm hypodense lesion 1cm deep to liver surface. Tests for hydatid disease
were -ve. Best course of action is?
A. Multiple aspirations,antiamebics and antibiotics
B. Catheter drainage with antiamebics and antibiotics
C .Hepatectomy(??)
d.. Medical management with antiamebics and antibiotics

Ans: ????????????

If the diagnosis is amebic liver abcess...then the TOC is medical ...and aspiration if
more than 5cm............in our question the abcess is 5cm .......

If pyogenic abcess then drainage...............are we considering it amebic only because it


is solitary?

50%pyogenic and 80% amebic abcess are solitary.................

If we are uncertain then the TOC should encompass both ---drain and give
antiamebics..................anyway stem shud be more clear......

The mainstay of treatment for amebic abscesses is metronidazole (750 mg orally three
times per day for 10 days), which is curative in more than 90% of patients. Clinical
improvement is usually seen within 3 days. Other nitroimidazoles (secnidazole,
tinidazole) are also as effective and are commonly used outside of the United States. If
response to metronidazole is poor or the drug is not tolerated, other agents can be used.
Emetine hydrochloride is effective against invasive amebiasis (particularly in the liver)
but requires intramuscular injections and has serious cardiac side effects. A more
attractive option is chloroquine, but this is a less effective agent. After treatment of the
liver abscess, it is recommended that luminal agents such as iodoquinol, paromomycin,
and diloxanide furoate are administered to treat the carrier state.[23]
Therapeutic needle aspiration of amebic abscesses has been proposed. Small
randomized trials comparing metronidazole with or without aspiration have shown minor
benefits with aspiration, but no major improvement to justify routine aspiration. In
general, aspiration is recommended for diagnostic uncertainty (see earlier), failure to
respond to metronidazole therapy in 3 to 5 days, or in abscesses felt to be at high risk
for rupture. Abscesses larger than 5 cm in diameter and in the left liver are thought to
be a higher risk for rupture, and aspiration needs to be considered

237. All are true about blood coagulation except?


A. Factor 10 in a part of both intrinsic and extrinsic pathway
B. Extrinsic pathway is activated by contact with plasma and negatively charged
proteins
C. Calcium is very important for coagulation
D. Intrinsic pathway can be activated in vitro

Ans: B. Extrinsic pathway is activated by contact with plasma and negatively


charged proteins

Reference:Robbins pathology 8th ed

Clinical laboratories assess the function of the two arms of the coagulation pathway
through two standard assays: prothrombin time (PT) and partial thromboplastin time
(PTT). The PT assay assesses the function of the proteins in the extrinsic pathway
(factors VII, X, II, V, and fibrinogen). This is accomplished by adding tissue factor and
phospholipids to citrated plasma (sodium citrate chelates calcium and prevents
spontaneous clotting). Coagulation is initiated by the addition of exogenous calcium and
the time for a fibrin clot to form is recorded. The partial thromboplastin time (PTT)
screens for the function of the proteins in the intrinsic pathway (factors XII, XI, IX, VIII,
X, V, II, and fibrinogen). In this assay, clotting is initiated through the addition of
negative charged particles (e.g., ground glass), which you will recall activates factor XII
(Hageman factor), phospholipids, and calcium, and the time to fibrin clot formation is
recorded.

238. Late onset endophthalmitis after lens implantation is caused by?


A. Staphylococcus epidermidis
B. Pseudomonas
C. Streptococcus pyogenes
D. Propionibacterium acnes

Ans:A Staph epidermidis/D Propionibacterium acnes

239. All are seen in the floor of 3rd ventricle except?


A. Infundibulum
B. Oculomotor nerve
C. Mammillary body
D. Optic stalk

Ans:B Occulomotor nerve

240. Which among the following is not a component of hypogastric sheath?


A. Broad ligament
B. Uterosacral ligament
C. lateral ligament
D. Ligament of bladder

Ans:A Broad ligamnent of uterus ---a peritoneal reflection

241. A patient presents with signs of pneumonia. The bacterium obtained from sputum
was gram positive cocci which grew on sheep agar. What test is used to identify the type
of organism?(Question stem incomplete---though it seemed like pneumococcal
pneumonia)
A. Bile solubility
B. Bacitracin sensitivity
C. Coagulase test
D. –

Ans:A Bile solubility

242. True about platelet function defect?


A. Normal platelet count with prolonged bleeding time
B. Thrombocytosis with prolonged bleeding time
C. --
D. --

Ans: A. Normal platelet count with prolonged bleeding time


243. After removal of pituitary for craniopharyngioma, first drug given is?
A.cortisone
B.GH
C. --
D. –

Ans:A Cortisone(no references though—will have to search)

244. A 5 year old child presented with ballooning of perpuce while micturition. Perpuce
adhesions were present. What is the best treatment for him?
A. Adhesiolysis and dilatation
B. Circumcision
C. Dorsal slit
D. Conservative

Ans:B Circumscision

245. Which of the following process in a vector is used to increase the yield of protein
produced in recombinant protein synthesis?
A. Promoter induction
B. Genes for protease inhibitors
C. Translation initiation
D. Translation and transcription termination

Ans:A Promoter induction(logically from these options---u need to increase


transcription and then only translation---mere increase in translation will not
increase protein output...so induce a promoter get more mrna and then more
protein via translation---hypothetical explanation----absolutely no
references...so subject to change in the light of better evidences)

246. VERSION 1:

Which of the following is not associated with atherosclerotic plaque formation?


A.Plasma ApoE
B. alpha 2-macroglobulin
C. Oxidised LDL
D. Increased homocystiene

Ans:B alpha 2 macroglobulin

Reference: Alpha2-antiplasmin is a single-chain glycoprotein serpin synthesized


predominantly by the liver. It is the main inhibitor of plasmin in human plasma, forming
a 1 : 1 stoichiometric complex with plasmin that inactivates the enzyme.[41] Alpha2-
macroglobulin also inhibits plasmin, but at a much slower rate than alpha2-antiplasmin;
therefore, alpha2-macroglobulin has questionable importance in the physiological
regulation of fibrinolysis

VERSION 2:

Increased levels of which of the following is not associated with atherosclerotic plaque
formation?
A.Plasma ApoE
B. alpha 2-macroglobulin
C. Oxidised LDL
D. Increased homocystiene

Ans:Both A and B

247. Least common cause of ambiguous genitalia in a female child?


A. Placental steroid sulfatase deficiency
B. Fetal aromatase deficiency
C. WT-4 mutation
D. CAH

Ans: A. Placental steroid sulfatase deficiency

248.A patient presented to the with bluish pigmentation of conjunctiva, mucous


membranes, nails. What is the poison:
A. Mercury
B. Arsenic
C. Lead
D. Silver

Ans:D Silver

249.ECG is poor at detecting ischaemia in areas supplied by?


A. Left anterior descending
B. Left circumflex
C. Left coronary artery
D. Right coronary artery

Posterior ischemia or an ischemia in the region of the left circumflex artery are often not
visible on a normal standard 12 lead ECG

ref-http://books.google.co.in/books?
id=MaDJ6nlaQKYC&lpg=PA235&pg=PA235#v=onepage&q&f=false

Ans-Left circumflex

250.Which of the following is a contraindication for medical treatment in gallstones?


A. Radio opaque stones
B. Radiolucent stones
C. Normal functioning gall bladder
D. Small stones

Bile acids help dissolve gall stones by decreasing the levels of cholesterol available for
stone formation
But radio opaque stones contain calcium in addition to cholesterol – dissolution of which
is not possible
Hence, bile acid therapy is contraindicated in radio opaque gall stones
ref-Diseases of the liver and biliary system By Sheila Sherlock, Sheila Sherlock (Dame.),
James Dooley

ans-radio opaque stone

251. Which of the following is not supplied by the anterior division of mandibular nerve
(V3) ?
A. Temporalis
B. Medial pterygoid
C. Lateral pterygoid
D. Masseter
Ans:B. Medial pterygoid

Reference: Wikipedia(shall search for better references)

Branches

Branches from the main trunk (except nervus spinosus) and the posterior division.

The mandibular nerve gives off the following branches:

 From the main trunk of the nerve (before the division)

 nervus spinosus (meningeal branch)

 medial pterygoid nerve

 nerve to tensor tympani

 nerve to tensor veli palatini

 From the anterior division

 masseteric nerve

 deep temporal nerves (anterior and posterior)

 buccal nerve (a sensory nerve)

 lateral pterygoid nerve

Branches from the posterior and anterior divisions (except lateral pterygoid nerve)

 From the posterior division

 auriculotemporal nerve

 lingual nerve

 inferior alveolar nerve


 motor branch to mylohyoid and anterior belly of digastric muscles (mylohyoid nerve)

The mandibular nerve also gives off branches to the otic ganglion

252. Drugs used in prophylaxis of migraine are all except?


A. Propranolol
B. Flunarizine
C. Topiramate
D. Levetiracetam
Ans:D. Levetiracetam

253.Most effective microbial killing system


A.Reactive oxygen species
B.Lysozymes
C.Defensin
D.
Ans:A.Reactive oxygen species

254. Slide fixing in pathology most commonly done by


A.formaldehyde
B.alcohol
C.picric acid
D.glutraldehyde
Ans:A.formaldehyde
Reference: A Manual of Histology, General Anatomy, Embryology & Genetics

“Formaldehyde is the most widely used fixative”

255. Ophtalmoplegic migraine..


A.Headache with reversible lose of ophthalmic nerve function
B.Recurrent transient 3rd nerve palsy after headache
C.
D.
Ans:B.Recurrent transient 3rd nerve palsy after headache (REPEAT)

256. Patient having pain in epigastrium which radiates to back, serum amylase is
normal, on x ray gall stone seen and pancrease appeares bulky..
A.Acute Pancreatitis
B.Acute cholecyctitis
C.Duodenal ulcer
D.
Ans:A.Acute Pancreatitis

257. During TURP, surgeon takes care to dissect above the verumontenum so as to
prevent injury to?
A. External urethral sphincter
B. Urethral crest
C. Prostatic utricle
D. trigone of bladder
Ans:A. External urethral sphincter

Reference: Reoperative Pelvic Surgery By Richard P. Billingham


“The veru montanum serves as the landmanrk in TURP as the External urethral sphincter
lies just distal to it.”

“It is crucial to limit resection from bladder neck to veru montanum”

Reference: Reoperative Pelvic Surgery By Richard P. Billingham


“The veru montanum serves as the landmanrk in TURP as the External urethral sphincter
lies just distal to it.”

“It is crucial to limit resection from bladder neck to veru montanum”

258. Which organ obtained from a cadaver is not used for transplantation?
A. Blood vessel
B. Lung
C. Liver
D. Bladder
Ans:D. Bladder

259. Baby born at 33 weeks / 1.5 kg should be started on?


A. Nil oral and IV fluids
B. Oral nasogastric tube/alternate oral route
C. IV fluids and oral feeding
D. TPN
Ans:B. Oral nasogastric tube/alternate oral route

Reference:Nelson textbook of Pediatrics

The method of feeding each LBW infant should be individualized. It is important to avoid
fatigue and aspiration of food by regurgitation or by the feeding process. No feeding
method averts these problems unless the person feeding the infant has been well trained
in the method. Oral feeding (nipple) should not be initiated or should be discontinued in
infants with respiratory distress, hypoxia, circulatory insufficiency, excessive secretions,
gagging, sepsis, central nervous system depression, severe immaturity, or signs of
serious illness.
These high-risk infants require parenteral nutrition or gavage feeding to supply calories,
fluid, and electrolytes. The process of oral alimentation requires, in addition to a strong
sucking effort, coordination of swallowing, epiglottal and uvular closure of the larynx and
nasal passages, and normal esophageal motility, a synchronized process that is usually
absent before 34 wk of gestation.
Preterm infants at 34 wk of gestation or more can often be fed by bottle or at the breast.
Because the effort of sucking is usually the limiting factor, breast-feeding is less likely to
succeed until the infant matures. Bottle-feeding of expressed breast milk may be a
temporary alternative. In bottle-feeding, effort may be reduced by use of special small,
soft nipples with large holes. Smaller or less vigorous infants should be fed by gavage: A
soft plastic tube with No. 5 French external and approximately 0.05 cm internal
diameters and with a rounded atraumatic tip and two holes on alternate sides is
preferable. The tube is passed through the nose until approximately 2.5 cm (1 inch) of
the lower end is in the stomach. The free end of the tube has an adapter into which the
tip of a syringe is fitted, and a measured amount of fluid is given by pump or by gravity.
Such tubes may be left in place for 3–7 days before being replaced by a similar tube
through the alternate nostril. Infants occasionally have enough local irritation from an
indwelling tube that they may gag or troublesome secretions may gather around it in the
nasopharynx. In such cases, a catheter may be passed through the mouth by a skilled
person and removed at the end of each feeding.
The LBW infant may be fed with intermittent bolus feedings or continuous feeding. In the
occasional infant with feeding intolerance, nasojejunal feeding may be successful.
Intestinal perforation is a risk with nasojejunal feeding. A change to breast- or bottle-
feeding may be instituted gradually as soon as an infant displays general vigor adequate
for oral feeding without fatigue.
Gastrostomy feeding is not usually indicated in premature infants except as an adjunct
to surgical management of specific gastrointestinal conditions or in permanently
neurologically injured patients unable to suck and swallow normally.
Initiation of Feeding.
The optimal time to introduce enteral feeding to a sick LBW infant is controversial.
Trophic feeding is the practice of feeding very small amounts of enteral nourishment to
VLBW preterm infants to stimulate development of the immature gastrointestinal tract.
The benefits of trophic feeding include enhanced gut motility, improved growth,
decreased need for parenteral nutrition, fewer episodes of sepsis, and shortened hospital
stays. Once the infant is stable, small-volume feedings are given in addition to
intravenous fluids/nutrition. Feeding is gradually advanced and parenteral nutrition
decreased. This approach may reduce the incidence of necrotizing enterocolitis. The
main principle in feeding premature infants is to proceed cautiously and gradually.
Careful early feeding of breast milk or formula tends to reduce the risk of hypoglycemia,
dehydration, and hyperbilirubinemia without the additional risk of aspiration, provided
that the presence of respiratory distress or other disorders does not present an
indication for withholding oral feedings and administering electrolytes, fluids, and
calories intravenously.
If an infant is well, is making sucking movements, and is in no distress, oral feeding may
be attempted, although most infants weighing <1,500 g require tube feeding because
they are unable to coordinate breathing, sucking, and swallowing. Intestinal tract
readiness for feeding may be determined by active bowel sounds, passage of meconium,
and the absence of abdominal distention, bilious gastric aspirates, or emesis. For infants
under 1,000 g, the initial feedings are either half- or full-strength breast milk or preterm
formula at 10 mL/kg/24 hr as a continuous nasogastric tube drip (or given by
intermittent gavage every 2–3 hr). If the initial feeding is tolerated, the volume is
increased by 10–15 mL/kg/24 hr. The daily milk volume increment should not exceed
20–30 mL/kg/24 hr. Once a volume of 150 mL/kg/24 hr has been achieved, the caloric
content may be increased to 24 or 27 kcal/oz. With high caloric density, infants are at
risk for dehydration, edema, lactose intolerance, diarrhea, flatus, and delayed gastric
emptying with emesis. Intravenous fluids are needed until feedings provide
approximately 120 mL/kg/24 hr. The feeding protocol for premature infants weighing
over 1,500 g is initiated at a volume of 20–25 mL/kg/24 hr of full-strength breast milk or
preterm formula given as a bolus every 3 hr. Thereafter, increments in total daily
formula volume should not exceed 20 mL/kg/24 hr. The expected weight increments for
premature infants of various birthweights are projected from Figure 97-6 . Infants with
IUGR may not demonstrate the initial weight loss noted in premature infants.
260.Which of the following drug is not used for medical treatment of diabetic
retinopathy?
A.Tamoxifen
B. --
C. --
D. --
Ans:A.Tamoxifen

261.Patient prestent with high TSH, low T4, what is the diagnosis?
A.Grave's disease
B.Hashimoto's disease
C.Pituitary failure
D.Hypothalamic failure

Ans:B.Hashimoto's disease (ruling out the others)

262.Pastient present with epigastric pain which radiates to the back and relieved by
food, patient have history of such pain in past for which he was taking analgesics and in
past 5 years 2 times operated for duodenal ulcer. What is the diagnosis
A. Gastric ulcer
B. Dudenal ulcer
C. Chronic pancreatitis
D.
Ans:Duodenal Ulcer

263. All are factors responsible for resurgence of malaria except?


A. Drug resistance
B. Use of bed nets
C. Vector resistance
D. Mutation in parasite
Ans:B. Use of bed nets (modified REPEAT)

264. What is false about lipoproteins.


A.HDL inhibit oxidation of LDL
B.Oxidize LDL not present in foam cells
C.
D.
Ans:B.Oxidize LDL not present in foam cells

265. 1yr old child present with growth failure,dry skin and palpable thyroid,with low
thyroid harmones and a high TSH, what is cause?
A.Dysharmonogenesis
B.Dysgenesis
C. Central Hypothyroidism
D. TSH Receptor mutation(?)
Ans:A Dysharmonogenesis(ruling out options)

266.Rise in end tidal CO2 during thyroid surgery can be due to all except:
A. Anaphylaxis
B. Malignant hyperthermia
C. Thyroid storm
D. Neuroleptic malignant syndrome
Ans:A. Anaphylaxis

Reference:

Conditions That Can Increase End-tidal

Carbon Dioxide

External causes

Alcohol therapy for limb arteriovenous malformation

Contrast dye

Drug toxicity or abuse

Environmental heat gain more than loss

Exercise hyperthermia

Heat stroke

Ventilation problems (kinked or blocked endotracheal tube)

Equipment malfunction (faulty expiratory valve)

Treatment of acidosis

Tourniquet release

Neuroleptic malignant syndrome

Carbon dioxide insufflation

Disease related

Cystinosis

Hypokalemic periodic paralysis

Intracranial free blood

Muscular dystrophies (Duchenne, Becker)

Central core disease

Myotonias
Diabetic coma

Freeman-Sheldon syndrome

Hyperthyroidism

Osteogenesis imperfecta

Pheochromocytoma

Prader-Willi syndrome

Rhabdomyolysis

Sepsis

King Denborough disease

Wolf-Hirschhorn syndrome

Idiopathic hyperCKemia

Malignant hyperthermia

267. All are true except:


A. Human anatomical waste is disposed in a yellow bag
B. Red bag contents can be a source of contamination
C. Black bag is used for incineration ash
D. Blue bag contents are always disposed in secure landfill

Ans:D. Blue bag contents are always disposed in secure landfill

268. Most common site of obstruction after TURP?


A. Navicullar foss
B. Bulb
C. Prostatic membranous urethra
D. Bladder neck
Ans:D. Bladder neck

269. Which of the following is true?


A. Acetylcholinesterase inhibition by malathion can be reversed by increasing the level of
atropine
B. Sulphonamide inhibits folate reductase irrevesibly
C. Flouoroacetate competetively inhibits aconitase
D. Ethanol inhibits aldehyde dehydrogenase when used in methanol poisoning

Ans:??????????????????
270. Small air way has laminar flow because?
A. Reynold number more than 2000
B. Diameter is very small
C. Extremely low velocities
D. Total cross sectional area low

Ans: lC. Extremely low velocities

Reference: Essentials of Pathophysiology: Concepts of Altered Health States


By Carol Mattson Porth, Glenn Matfin

http://books.google.com/books?
id=FFg88IaReBwC&pg=PA525&dq=laminar+flow+in+small+airways+cause&hl=en&ei=
BMQ2TeyqA8iPcYyrqJkC&sa=X&oi=book_result&ct=result&resnum=2&ved=0CDIQ6AEw
AQ#v=onepage&q=laminar%20flow%20in%20small%20airways%20cause&f=false

271. A neonate delivered at 38 weeks of gestation, birth weight of 2.2kg develops


intolerance to feeds on 2nd day. Physical examination reveals no abnormalities. Sepsis
screen in negative. And PCV is 70% What is the next step in management? (corrected by
dr_sid4u)
A. IV fluid
B. Presumptive trearment of sepsis
C. Exchange transfusion
D. Repeat sepsis screen
Ans:C Exchange transfusion

http://www.newbornwhocc.org/pdf/Polycythemia_2010_200810.pdf

Management of polycythemia is dependent upon two factors (Figure):

1. Presence of symptoms suggestive of polycythemia and/or

2. Absolute value of hematocrit

(a) Symptomatic polycythemia

The definitive treatment for polycythemia is to perform a partial exchange transfusion

(PET). PET involves removing some of the blood volume and replacing it with fluids so

as to decrease the hematocrit to a target packed cell volume of 55%.

272.Not a part of national screening program?


A. Diabetes mellitus
B. Dental caries
C. Refractive error
D. Carcinoma cervix
Ans.?????????????????

273.All are true regarding phagocytosis by protozoa except?


A. Amoeba n other unicellular org make their living out of it
B. Phagocytose particles of <0. 5 microns size
C. Phagocytose particles of > 0.5 microns size
D. Digestion occurs within phagolysosomes
Ans:B. Phagocytose particles of <0. 5 microns size

274. In L5 root involvement, which among the following is not affected?


A. Thigh adduction
B. Knee flexion
C. Knee extension
D. Toe extension

Ans:C Knee extension

Quadratus femoris also has a contribution in thigh adduction and L5 is among


the root values of this
muscle.......... https://www.msu.edu/user/vosskurt/Miscellaneous
%20pages/musnvrt.htm

275. 14.A sewer worker presented with fever. Lab findings revealed renal failure with
increased BUN and serum creatinine. What is the most appropriate drug to give him?
A. Cotrimoxazole
B. Erythromycin
C. Ciprofloxacin
D.Benzyl penicillin

Ans: D Benzyl penicillin

Reference : CMDT 2011 and Harrison principles of Internal Medicine 17th ed

The Case is that of Leptospirosis:

The organism may also enter through minor skin lesions and probably via the
conjunctiva. Recreational cases have followed swimming or rafting in contaminated
water, and occupational cases occur *****among sewer workers*****, rice planters,
abattoir workers, and farmers. Sporadic urban cases have been seen in the homeless
exposed to rat urine. The incubation period is 2–20 days.

. ***Penicillin (eg, 1.5 million units every 6 hours intravenously)**** or


ceftriaxone (1 g daily intravenously) is the drug of choice in severe leptospirosis and is
especially effective if started within the first 4 days of illness.

*****Penicillin can be given in renal failure with dose


modification*****….Though erythromycin which was given in the options has also been
mentioned as a treatment for severe leptospirosis in Harrison along with
Penicillin….Benzyl Penicillin G remains the Drug of Choice for severe leptospirosis…

276. All are true about ranalozine except?


A. Causes hypotension
B. It is recommended as first line treatment for Angina
C. Improves glycemic control
D. It is an anti angina drug
Ans: B It is recommended as a first line treatment of Angina.

Option D is correct

Option A is Also correct---It does cause postural hypotension and hypotension too(as
previously pointed out by many ---FDA site mentions it)

Adverse effects

The most common adverse effects are dizziness, nausea, asthenia and constipation.
Postural hypotension, syncope, headache, dyspepsia and abdominal pain are also
reported. An increase in QT interval was observed at doses of 750 and c1000 mg b.i.d.
However, torsades de pointes is not reported.

Option C is also correct

Now Option B:

Ranolazine was introduced in 2006 as a drug to be used in *****chronic angina*****


along with other mainstay drugs….In 2008 it seems to have been given the green signal
to be used as a first line agent in ****chronic angina****

This option shall go wrong by 2 ways:

1.Latest editions of standard Cardiology books like Braunwald /Hurst and even Herry all
mention it to be used along with other agents..

2.At the present time too the indication is only for CHRONIC angina----the option clearly
mentioned ANGINA…..so assuming it to be chronic angina and taking it as a right option
when the other three are right ….will be a little too odd….

****So, We feel the answer should be B.Its not a first line management for ANGINA
overall though it may be for CHRONIC…****

CHEERS!!!

277. A lady who presented with hematuria on evaluation was found to have stage 2
transweritional cell carcinoma of bladder. Which of the following is true?
A. 70% chance of requiring cystectomy in 5 yrs
B. Cystoscopic fulguration will have to be done
C. A 10 year history of beedi smoking is not a risk factor
D. There is no chemotherapy available

Ans:A . 70% chance of requiring cystectomy in 5 yrs

Reference:Shwartz Surgery:

Bladder Cancer
The most common form of bladder cancer in the United States is transitional cell
carcinoma (TCC). ******Tobacco use(OPTION C), followed by occupational exposure to
various carcinogenic materials such as automobile exhaust or industrial solvents are the
***most frequent*** risk factors, though many with the disease have no identifiable
risks.
Management of TCC varies greatly, depending on the depth of invasion. A
complete transurethral resection of the bladder tumor, which allows staging of the
tumor, is the first step.
For patients that have disease invading into *****bladder muscle
(T2)****(OPTION A over B), *****immediate cystectomy with extended lymph node
dissection**** offers the best chance of survival, although current long-term cure for
those presenting with clinically localized disease is still only achieved in 50 to 60% of
patients.
The *****addition of neoadjuvant or adjuvant chemotherapy in those without
discernible metastatic spread is gaining increasing acceptance*****OPTION D and does
provide an increase in survival.4 Patients with limited lymph node involvement may be
cured with surgery alone, but those with extensive lymph node involvement have a
dismal prognosis.
278.Which is not seen in digoxin toxicity?
A. Biventricular tachycardia
B. atrial tachycardia with variable AV block
C. Ventricular bigeminy
D. Regularisation of AF

Ans: NONE (is it nessasary to always get an answer to a question or what ??....lol……
******if these were the exact options and the stem**** then all are found in Digitalis
toxicity as per books….though biventricular tachy is not mentioned everywhere…..)

Reference: I am quoting the lines of Irwin and Rippe's intensive care medicine

As u may see …..its a line to line pick…………..also found in many other books though

Dysrythmias frequently associated with Cardiac Glycoside toxicity are:

Premature ventricular contraction

****Paroxysmal atrial tachycardia with block****

AF with junctional block

Sinus bradycardia

Ventricular tachycardia

******Ventricular Bigemini******OPTION C

*****Atrial tachycardia with variable AV block*****OPTION B

******Atrial Fibrillation with accelerated or slow junctional rhythm(regularisation of


AF)*******OPTION D

******Bidirectional ventricular tachycardia---most characteristic arrhythmia******

Biventricular tachycardia---- ******OPTION A


279. A 6 year old child presents with pain and tenderness in hip in femoral triangle
region.limitation of movements, X-ray does not reveal any abnormality. What is the next
step?
A. USG
B. MRI
C. Aspiration
D. Wait and watch

Ans:Best response shall be MRI that is B

Assuming the age of the child and the limited details provided few DD’s do come up..Of
which Leg calve perthe’s disease is better…Now these are the lines from LANGE
Radiology and GRAINGER Radiology….

Radiography is a rather insensitive *****(FITS IN OUR CASE)diagnostic tool for


evaluating the early manifestations of osteonecrosis. Osteonecrosis does cause zones of
increased sclerosis and osteolysis in the affected bone that can be appreciated on
radiographs. However, these changes are late and early treatment cannot be performed
at this stage (Fig. 7–49). If the disease is not diagnosed and treated early, the affected
bone may go through a phase of subchondral collapse and become deformed.
Subsequently, secondary osteoarthrosis will develop in the affected joint as a
complication of neglected or poorly treated disease. Traditionally, nuclear medicine bone
scanning has been used in this setting,******* but today MR imaging is the most
sensitive******* available modality for the early diagnosis of this disease

Though USG can also be done in this case…..MRI being widely available and good
enough to pick up early cases wud be the better choice…..

280. All are true about world health report 2008 except?
A. Social reforms
B. Leadership reforms
C. Polices reforms
D. Economic reforms

Ans:D Economic reforms

Ref:WORLD HEALTH REPORT 2008


http://www.who.int/whr/2008/08_contents_en.pdf

Sorry I could not cut paste.But here's exact words from it.

4sets of PHC reforms:


Solidarity and social inclusions:Universal coverage reforms
People centred care:Service reforms
Health authorities:Leadership reforms
communities where health is promoted and protected:Public policy reforms
Here social reforms are mentioned under universal coverage reforms.Nowhere does it
mention economic reforms.

About universal coverage reforms it mentions:reforms that ensure that health systems
contribute to health equity,social justice and the end of exclusion,primarily by moving
towards universal access and social health protection.
--

281. Regarding Clostridium tetani, all are true except?

A. Spores are resistant to heat


B. 3 doses give immunity in primary immunisation
C.Incubation period is 6-10 days
D. Person to person transmission does not occur

Ans: B 3 doses give immunity in primary immunisation

OPTION A /C/D correct as per park and Harrison Medicine…

For tetanus per se it is only 2 doses in primary immunization ….though as a part of DPT
in the NIS it is given as 3 doses followed by 2 boosters…..

Version 2::::many of the doctors feel the first option was The organism is heat
resistant…..in which case it becomes a better answer considering the possibility of the
examiner meaning the NIS schedule primary doses…..so these are the possibilities---

282.A child presents with abdominal pain only during passage of stools. No other
symptoms like vomiting or blood in stools. There are no signs of intestinal obstruction.
Most probable diagnosis is?
A. Rectal polyp
B. Intusseception
C. Meckels diverticulum
D. NEC

Ans: Most probable answer A Rectal polyp>Intususseption


Actually speaking ****none of the options seem to fit in the case scenario
exactly*****….well we cant expect exact matches in medicine though…..

above presentation though not seen in acute intussuseption can be seen in chronic
intussuseption which often presents a diagnostic difficulty….

Chronic Intussuseption is mostly present with episodes of non-specific abdominal pain of


prolonged duration from two weeks to three years. This is considered by some authors
as the only reliable clinical feature in establishing the diagnosis….

BUT the pathophysiology of intussusceptions is such that----ABDOMINAL PAIN will be


*****followed BY***** passage of stools(not accompanying the stools usually) which
are usually bloody---not always though……..

Rectal Polyp can have a presentation of pain on passing stools due to the *****traction
of the polyp while passing stools.******(the polyp is pushed into the anus while the
child strains)……Though Rectal polyp most commonly presents with BLEEDING……..I feel
this Q might be a twister……

CHEERS!!!

283. True regarding chlamydia trachomatis is?


A. Culture of purulent endocervical discharge is used for isolation of organism
B. Patient using OCP's are carriers
C.--
D. –

Ans: B. Patient using OCP's are carriers

Reference:BEREK and NOVAK Gynecology

Chlamydial colonization of the cervix appears more likely in OC users than in nonusers
but,
despite this finding, several case control studies have found a reduced risk of acute
pelvic inflammatory disease among OC users (163,164). In contrast, a recent study
found
no protection with OC use (165).*****OPTION B

Diagnosis
The diagnosis of cervicitis is based on the finding of a purulent endocervical
discharge, generally yellow or green in color and referred to as “mucopus” (30).

After removal of ectocervical secretions with a large swab, a small cotton swab
is
placed into the endocervical canal and the cervical mucus is extracted. The
cotton swab
is inspected against a white or black background to detect the green or yellow
color of
the mucopus. In addition, the zone of ectopy (glandular epithelium) is friable or easily
induced
to bleed. This characteristic can be assessed by touching the ectropion with a cotton
swab or spatula.

Placement of the mucopus on a slide that can be Gram stained will reveal the
presence
of an increased number of neutrophils (30 per high-power field). The presence
of intracellular gram-negative diplococci, leading to the presumptive diagnosis of
gonococcal endocervicitis, also may be detected. If the Gram stain results are negative
for gonococci, the presumptive diagnosis is chlamydial cervicitis.
P.549

Tests for both gonorrhea and chlamydia, preferably using nuclei acid
amplification
tests, should be performed.******(OPTION A ruled out) The microbial etiology of
endocervicitis is unknown in about 50%
of cases in which neither gonococci nor chlamydia is detected.

284.A 5 yr old boy presented with leukocoria in right eye ball, while other eye had 2-3
small lesions in the periphery. What will be the ideal management for this patient?
A. Enucleation of both eyes
B. Enucleation of right eye & conservative management for the other eye
C. Enucleation for right eye and radiotherapy for the other eye
D. 6 cycles of chemotherapy

Ans: D. 6 cycles of chemotherapy

OPTION B was being performed previously…now chemoreduction is being performed for


bilateral advanced disease..

Reference: Ophthalmic Oncology By Bita Esmaeli

285. A patient presented with an abdominal injury with peritonitis and shock. Airway,
breathing and IV fluids for circulation were taken care of. What is the next step of
management?
A. Take the patient for laparotomy under GA
B. Take the patient go for a laparoscopy
C. Insert an abdominal drain under LA and take up for immediate surgery
D. correct electrolyte abnormalities and take patient to OT

Ans: D. correct electrolyte abnormalities and take patient to OT

286.Two plants are grown. One genetically endowed with ability to produce a green
fluorescent pigment and the other with firefly luciferase.Which plant will glow in the
dark?
A. Both plants will glow
B. Neither will glow
C. First one will glow
D. Second one will glow

Ans: B. Neither will glow

Explanation----If these were the exact wordings of the question then tte answer is
C…..fluorescent pigment exhibits tthephenomenon of fluorescence in the dark by
itself…

In case of lucifeerasse the plant will not grow since it also needs the substrate
LUCIFERIN on which it acts produce bochemiluminiscence….

287.Mr X is a chronic smoker. His family insists on quitting smoking. He is thinking about
quitting, but is reluctant to do so because he is worried that on quitting he will become
irritable. This is?
A. Precontemplation and preparation
B. Contemplation and extent of sickness susceptability
C. Contemplation and cost factors

D. Precontemplation and cost factors

Ans:C Contemplation and cost factors(if this was the correct Q)

Contemplation since the person is thinking about quitting and cost factors---the effects
he may have to stand after quitting

288.Which of the following is Contra indicated in a patient with seizures/epilepsy?

A. OCP
B. POP
C. IUCD
D. –

Ans:A OCP---Reference Park

289.8 yr old with sign of meningitis and ear infections with ring enhancement in the
fronto temporal lobe all can cause it except
A.H.infuenzeae
B.staphylococcus
C.pneumococcus
D.pseudomonas

Ans:A H.influenzae

Reference:ENT by Dr.Dhingra

This is a case of cerebral abcess secondary to otitis media Dhingra mentions B/C/D as
common causes…and mentions H.influenzae as RARELY SEEN…and hence the answer..
290.Vectors don't transmit infection by?
A. Ingestion
B. Regurgitation
C. Rubbing of feces
D. Contamination with body fluids

Ans: A Ingestion

Reference: Park

291.A investigator finds out that 5 independent factors influence the occurrence of a
disease. Comprision of multiple factors responsible for a disease can be assessed by?
A. ANOVA
B multiple linear regression
C. Kruskal-wallis analysis of ranks
D. multiple logistic regression

Ans:B Multiple linear regression

292.About diabetes insipidus all true except


A.Water deprivation test is diagnostic.
B.Before doing test first correct hypoaldosteronism.
C.Hypothyroidism not affect the diagnostic test.
D.Pre test serum osmolarity >288, before giving vasopressin.

Ans:C Hypothyroidism not affect the diagnostic test. (????????????)

293.All are non deleberate measures for control of mosquito except?


A. Use of alkaline soap water in factory
B. Use of larvicidal agents
C. Community participation
D. use of bed nets for mosquito

Ans: A. Use of alkaline soap water in factory

294.In superior oblique palsy diplopia is :


A.horizontal and down
B.horizontal and up
C.vertical and dwn
D.vertical and up

Ans: C.Vertical and down

Explanation:Reference: Neurology in Clinical Practice: Principles of diagnosis and


management
By Walter George Bradley

In superior oblique palsy the most common diplopia is Isolated vertical diplopia
especially on looking down…..

Horizontal diplopia may also be found…

295.Ttrue about prion


a.catalyses folding of other proteins

b.—

c.—

d.—

Ans:A Catalyses the folding of other proteins

296. Methods of fusing two cells in genetic recombination techniques are all except?
A. Attaching inactive viral particle on cell membrane
B. Adding ethylene glycol
C. Applying a small electric current
D. Reducing the viscosity of the membrane

Ans: A Attaching inactive viral particle on cell membrane(most probably---References


awaited)

297. Which among the following does not cause hyperpyrexia?


A. MAOI
B. Alcohol
C. atropine
D. Amphetamine
Ans: B Alcohol
Reference:Harisson 17th ed
Alcohol(ethanol) causes hypothermia and not hyperthermia
298. Arthropod transmitted disease not found in India?
A. West nile fever
B. Dengue
C. Yellow fever
D. –

Ans:C Yellow fever

“”””If this Q was there”””


299.Direct standardisation is used to compare the mortality rates between two
countries.This is done because of the difference in:

A.Causes of death

B.Numerators

C.Age distribution

D.Denominators

Ans:C Age distributions(REPEAT)

300. Which among the following is preferred in a patient with decreased renal functio to
avoid contrast nephropathy?
A. N acetylcysteine
B. Fenoldopam
C. Low osmolar contrast
D. Mannitol
Ans: A

Das könnte Ihnen auch gefallen